„Offensichtliche“ Theoreme, die eigentlich falsch sind

Es ist einer der Lieblingssprüche meines Professors für echte Analyse, dass "offensichtlich zu sein bedeutet nicht, dass es wahr ist".

Nun, ich kenne einige Beispiele für Dinge, die offensichtlich wahr sind und deren Wahrheit bewiesen werden kann (wie das Jordan-Kurven-Theorem).

Aber was sind einige Theoreme (vorzugsweise kurze), von denen der Durchschnittsmensch behaupten würde, wenn sie in Laiensprache ausgedrückt werden, dass sie wahr sind, die aber tatsächlich falsch sind (dh kontraintuitiv falsche Theoreme)?

Die einzigen, die mir einfallen, sind das Monty-Hall-Problem und die Divergenz von N = 1 1 N (Zumindest für mich kontraintuitiv, da 1 N 0 ).

Das vermute ich auch

lim N ( 1 + 1 N ) N = e = N = 0 1 N !
ist nicht offensichtlich, da man das 'erwartet' ( 1 + 1 N ) N ( 1 + 0 ) N = 1 .

Ich suche nur nach Theoremen und nicht nach deren (Wider-)Beweis - ich recherchiere das gerne selbst.

Danke!

Diese Frage ist schwer zu beantworten, denn eine wesentliche Fähigkeit für einen Mathematiker besteht darin, seine Intuition schnell an die Wahrheit anzupassen. Sobald Sie also wissen, dass eine Tatsache falsch ist, erscheint sie sehr bald nicht mehr offensichtlich.
@NateEldredge Ich stimme zu, aber es gibt immer jemanden, der sich weigert, das zu akzeptieren, zum Beispiel, 0,999... = 1 .
Jeder kompakte Satz ist Jordan-messbar (false): math.stackexchange.com/questions/485336/…
Ich widerspreche entschieden, dass das Jordan-Kurven-Theorem offensichtlich wahr ist! Ich würde zustimmen, dass das Jordan Curve Theorem für stückweise glatte Kurven offensichtlich wahr ist; aber es ist auch ziemlich einfach zu beweisen, zumindest im Vergleich zur topologischen Version. (Ist es wirklich offensichtlich, dass die Koch-Schneeflocke keinen pathologischen Weg von innen nach außen hat?)
Vielleicht möchten einige Leser zu jeder Antwort Kommentare darüber abgeben, wie sie ihre Intuition so abgestimmt haben, dass das Thema der Antwort offensichtlich wird (ich meine, die Falschheit offensichtlich machen). Solche Kommentare wären ebenso nützlich wie die Antwort selbst. Nehmen wir zum Beispiel das Monty-Hall-Problem, bei dem sich die erste naive Vermutung durch eine brachiale Fall-für-Fall-Analyse als falsch herausstellt. Eine abgestimmte Intuition könnte aus jeder Erklärung gewonnen werden, die keine Brute-Force-Analyse verwendet, zum Beispiel könnte man die Bayes-Regel verwenden
"Jede wahre Aussage kann bewiesen werden"
@sinelaw Interessant. Können Sie ein Gegenbeispiel für eine solche Behauptung finden (wenn es wahr ist und wir es wissen, dann kann es bewiesen werden, aber wenn es wahr ist und wir es nicht wissen, dann ist es nicht unbedingt wahr)?
@alexqwx, es brauchte Gödel, um das zu widerlegen ... es ist sein erstes Unvollständigkeitstheorem ( en.wikipedia.org/wiki/G%C3%B6del%27s_incompleteness_theorems )
@sinelaw, OK, bitte liefern Sie einen formellen Beweis für Ihre Aussage.
@sinelaw Ich glaube, Sie haben die Bedingungen für Gödels Theoreme verpasst. Er hat keine Behauptung über alle formalen Systeme aufgestellt, und hätte er dies getan, hätte Goedel den Vollständigkeitssatz für Prädikatenkalkül erster Ordnung widerlegt, den er zuvor bewiesen hatte (ich denke, Hilbert-Bernays 'Axiomisierung eines solchen).
Ich stimme dafür, diese Frage als nicht zum Thema gehörend zu schließen, da es bereits genug offensichtliche Theoreme gibt

Antworten (71)

Satz (falsch):

Man kann die Terme in einer konvergenten Reihe beliebig neu anordnen, ohne ihren Wert zu ändern.

Haben Sie ein explizites Gegenbeispiel zu dieser Tatsache?
@DanielRobert-Nicoud (Bedingt) konvergente Reihe. =)
Die Serie ( 1 ) ich ich ist ein Gegenbeispiel; siehe diesen Wikipedia-Artikel für eine Diskussion.
@MJD Danke!
@OL Ich weiß (jetzt), dass das Neuanordnen der Begriffe zu einem falschen Ergebnis führt. zB das ln ( 2 ) = 1 2 ln ( 2 ) , aber wo liegt der Irrtum bei der Neuordnung der Begriffe?
@alexqwx Grob gesagt, dass einige Terme "unendlich weit weg" geschoben werden können, sodass sich zwei beliebige endliche Teilsummen um beliebig viele Terme unterscheiden. Stellen Sie sich zB vor, Sie nehmen die positiven Zahlen und ordnen sie als 1, 2, 3, 5, 4, 7, 9, 11, 6, 13, 15 usw. mit einer wachsenden Anzahl ungerader Terme zwischen jeweils zwei aufeinanderfolgenden geraden Termen an; dann taucht jede Zahl irgendwann immer noch in der Folge auf, aber das bedeutet nicht, dass irgendein Konzept wie ein 'Verhältnis' von ungerade zu gerade (zB natürliche Dichte ) erhalten bleibt.
@DanielRobert-Nicoud: Da es Gegenbeispiele gibt, ist es keine Tatsache ...
Häh? Man kann die Terme in jeder Reihe neu anordnen, konvergent oder anders. Hast du bei der Aussage deines Theorems etwas ausgelassen?
@MJD Diese Reihe ist kein Gegenbeispiel zu der Aussage "Man kann die Terme in einer konvergenten Reihe neu anordnen"; Tatsächlich enthält der von Ihnen zitierte Wikipedia-Artikel mehrere explizite Beispiele für Neuanordnungen dieser Serie.
Tatsächlich können Sie Terme in einer konvergenten Reihe neu anordnen, wenn Sie ihren Wert berechnen (oder auf andere Weise). Mit ein wenig Sorgfalt können Sie sogar Terme neu anordnen, ohne den Wert zu ändern. Was Sie nicht annehmen können, ist, dass jede Neuanordnung die Konvergenz beibehält, oder wenn sie den konvergierten Wert beibehält.
@alexqwx Es gibt keinen Irrtum. Was passiert, ist, dass die übliche Definition von Reihen in Bezug auf eine ganz bestimmte Art und Weise gegeben wird, Werte zu erhalten, die nicht immun gegen Neuordnung ist.
@MarcvanLeeuwen richtig, das kann ich sagen N = 0 N = 1 12 , aber für die meisten Situationen ist dies nicht hilfreich und nicht wahr.
Das klingt ziemlich kaputt, steht das nicht im Widerspruch zur Kommutativität der Addition?
@kutschkem Sie können immer endlich viele Swaps von Begriffen (unter Verwendung der Kommutativität) machen, ohne den Wert zu ändern, aber Sie können nicht unendlich viele Swaps machen; Kontinuität wird nicht über eine unendliche Anzahl von Operationen hinweg beibehalten.
@StevenStadnicki Ok, ich glaube, ich verstehe, können Sie mir irgendwo zeigen, wo dies explizit gemacht wird (in dem verlinkten Wikipedia-Artikel wird ein Gegenbeispiel erstellt, aber nichts darüber gesagt, dass "Kontinuität nicht über eine unendliche Anzahl von Operationen hinweg erhalten bleibt ".
Wenn eine Reihe nicht absolut konvergiert, gibt es Teilfolgen zu Und . Dass es dann bei Nachbestellungen nicht gleichförmig wird, dürfte den Kennern von Hilberts Hotel klar sein.
Liegt das Problem nicht darin, wie wir Serien analysieren, nicht in harmlosen Transformationen?
Warum wird diese Antwort akzeptiert? Es scheint, dass aufgrund der Art der Frage keiner sein sollte.
@bof bedeutet eindeutig "Angenommen, wir haben eine konvergente Reihe mit der Summe S. Wir können die Terme beliebig neu anordnen, und die Reihe konvergiert immer noch gegen die Summe S."
@MattMcNabb Und mein Kommentar bedeutet eindeutig "Ich weiß, was du sagen wolltest, aber du hast es nicht gesagt."
@Ollie Wahrscheinlich ist es derjenige, den das OP am meisten mag. Es gibt keine Möglichkeit, eine Frage für eine akzeptierte Antwort unzulässig zu machen.
Dieser ist besonders amüsant. Wenn Sie dies auf WAHR ändern und die mathematischen Regeln für die Unendlichkeit ändern, um diese Änderung zuzulassen, gilt der fundamentale Satz der Analysis. Ich kann nicht beweisen, dass das Ergebnis vollständig konsistent ist, aber es scheint gut zu funktionieren. Damit entfällt auch der fiese Biss aus dem Axiom of Choice (die Verdopplung der Sphäre gilt nicht mehr).
Ich habe dies bearbeitet, um zu sagen, was eigentlich gemeint war. Das hat mich beim letzten Mal gestört, aber ich habe beschlossen, es nicht zu beheben.
Wenn das wahr ist, dann ist es so seltsam, dass ich Konvergenz fast neu definieren möchte, um es anders zu machen.
Ich habe das auf die harte Tour gelernt.
@mathreadler Keine Notwendigkeit, Konvergenz neu zu definieren; Es gibt bereits absolute Konvergenz , was genau erforderlich ist, um willkürliche Umordnungen zu ermöglichen.

Eine Form mit endlichem Volumen muss eine endliche Oberfläche haben.

Dies ist offensichtlich falsch. Nehmen Sie einen Tonzylinder mit Durchmesser 1. Rollen Sie ihn in Ihren Händen, so dass sein Durchmesser wird 1 2 . Das Volumen ist gleich, aber er ist jetzt viermal so lang wie zuvor, hat also die doppelte Oberfläche des ursprünglichen Zylinders. (Halber Umfang, aber vierfache Länge.) Etwas weiter rollen, und die Oberfläche vergrößert sich wieder. Indem Sie die Schlange sehr lang und dünn machen, können Sie die Oberfläche bis ins Unendliche vergrößern, während das Volumen konstant bleibt. Das ist nicht nur offensichtlich, es ist alltäglich.
Das bekannte Beispiel eines Schwamms veranschaulicht eine andere Möglichkeit, dass ein Festkörper ein endliches Volumen, aber eine große oder potenziell unendliche Oberfläche haben könnte.
Das sind gute Beispiele. Ich weiß, dass dies niemanden verwirren würde, der wie die meisten Poster hier formell ausgebildet ist, aber ich habe versucht, ein Beispiel für "Laienbegriffe, die durchschnittliche Person" vorzuschlagen, wie von OP gefordert. Es schien, als wären die meisten der bisherigen Beispiele nicht einmal für eine ungeschulte Person interpretierbar.
@MJD: Dein Zylinderbeispiel ist nett, aber es zeigt nur, dass die Oberfläche beliebig groß sein kann. Das OP weist darauf hin, dass die Oberfläche buchstäblich unendlich sein kann.
Ja, aber um es tatsächlich unendlich zu machen, folgen Sie dem gleichen Prinzip und verwenden eine Schlange, die an einem Ende dick ist, aber dünner wird, wenn Sie ins Unendliche gehen. Dann ist das einzige Überraschende, dass diese unendlich lange Schlange ein endliches Volumen haben kann. Aber wir haben diese Kuriosität vor langer Zeit geschluckt; Ich habe es an anderer Stelle in diesem Thread zusammen mit Zenos Paradoxon erwähnt.
Das heißt, man rollt nicht die ganze Schlange, sondern nur die rechte Hälfte im Wachschritt.
@MJD: Wenn Sie die Oberfläche des Tons weiter erweitern, wird es dann nicht einen Punkt erreichen, an dem der Abstand zwischen den Atomen groß genug ist, um den Ton aufzulösen? Wenn das der Fall wäre, sollten Sie die Oberfläche eines Tons nicht unendlich erweitern können (ich weiß nicht, ich bin kein Mathematiker).
Ich weiß es auch nicht, ich bin ein Physik-Ignorant. Bist du sicher, dass Ton aus Atomen besteht?
Mathematisch mag das stimmen, aber physikalisch unmöglich, da Materie diskret ist. Die Grenze wäre eine Reihe von Atomen.
Menger Schwamm! 0 Volumen Unendliche Oberfläche!
Sie könnten eine Obergrenze festlegen, wenn Sie die Form auf konvex beschränken würden.
"Eine Form mit endlichem Volumen hat eine genau definierte (endliche oder unendliche) Oberfläche" ist weniger ehrgeizig, aber wahrscheinlich auch falsch (es sei denn, der Begriff Form hat eine eingeschränkte Bedeutung, die mir hier nicht bekannt ist).
Auf die gleiche Weise, aber möglicherweise weniger allgemein anerkannt, kann ein Objekt mit endlicher Masse (mathematisch gesprochen) keinen endlichen Schwerpunkt haben.
In ähnlicher Weise muss für 2D "eine Figur mit endlicher Fläche einen endlichen Umfang haben". Eine Darstellung des Mandelbrot-Sets beweist, dass es falsch ist: endliche Fläche, unendlicher Umfang.
Ich bin kein Mathematiker, nicht einmal annähernd. Aber könnte man das nicht anhand des V/A-Verhältnisses beweisen? Im Falle eines Zylinders denke ich, dass es h / (h + r) ist.
Nicht gut in Mathematik oder Physik, aber selbst wenn wir einen rein mathematischen Zylinder nehmen, wenn Sie ihn bis ins Unendliche dehnen, um zu beweisen, dass seine Oberfläche so ist, wird das Volumen nicht genau im selben Moment unendlich?
Koch-Tetraeder . Da ist dein Beweis.
@ deed02392 Nein, tut es nicht, aber diese Frage ist genau der Grund, warum das 'Ton-Gegenbeispiel' nicht offensichtlich ist.
Ich bin traurig, dass meine bisher am besten bewertete Antwort eine Aussage ist, die völlig falsch ist.
Ich verstehe immer noch nicht, wie diese Aussage falsch sein kann. Da „Unendlichkeit“ nicht existiert und nur ein mathematisches Konstrukt ist, um unser Verständnis der Natur zu unterstützen, scheint mir, dass alles Endliche tatsächlich endlich ist.
Gabriels Horn ist auch ein Beispiel. Lautstärke von π mit unendlicher Oberfläche.
@ user3932000: Das Schöne an Gabriels Horn ist, dass es eine so einfache Form hat, ein Rotationskörper, der als solcher konvexe Querschnitte in geeigneten Ebenen hat. Das macht es viel überraschender als Dinge wie Schwämme oder gestreckte Zylinder.
Das ist so, als würde man argumentieren, dass eine gerade Linie eine Fläche oder eine Ebene ein Volumen hat.

Vermuten A Und B spielen folgendes Spiel: A wählt zwei verschiedene Nummern über eine unbekannte Methode B , schreibt sie auf Zettel und steckt die Zettel in einen Hut. B zieht zufällig einen der Zettel und prüft seine Nummer. Sie sagt dann voraus, ob es die größere der beiden Zahlen ist.

Wenn B wirft einfach eine Münze, um ihre Vorhersage zu treffen, sie wird die Hälfte der Zeit richtig liegen.

Offensichtlich gibt es keine Methode, die besser abschneiden kann als der Münzwurf.

Aber es gibt ein solches Verfahren, beschrieben in Thomas M. CoverPick the largest number “ ( Open Problems in Communication and Computation Springer-Verlag, 1987, p152), das ich hier kurz und hier ausführlich beschrieben habe .

Gibt es wirklich ein durchgängiges Konzept einer gleichmäßigen Verteilung von N oder R + ? Ich könnte eine Verteilung wie verstehen e X Erzeugung einer zufälligen reellen Zahl aus 0 Zu , aber was ist mit einer Gleichverteilung?
Wer hat was von einer gleichmäßigen Verteilung gesagt N ? Sicherlich nicht ich.
Die Verteilung, die Person A wählt, bleibt also unbestimmt?
Ja. Das war meine Absicht, indem ich sagte, dass die Methode unbekannt sei B , das ist alles, was hier wichtig ist. Wie könnte ich das klarer machen?
Entschuldigung, ich sollte das hier nicht ansprechen, ich werde Ihren Link so gut wie möglich lesen und Ihren anderen Beitrag kommentieren, wenn ich muss, da ich denke, dass dies ein besserer Ort für Diskussionen ist. Ich stimme zu, zwei Laien, die dieses Spiel spielen, würden auf die Frage "Gibt es einen besseren Weg als einen Münzwurf" sicherlich nein sagen, also +1 unter der Annahme, dass Ihr Ergebnis korrekt ist ^_^ Ich denke, ich habe nur Probleme, Mechanik zu assoziieren Prozesse zu nicht berechenbaren Konzepten (wie die Auswahl einer zufälligen reellen Zahl).
Dies könnte vollständig berechenbar gemacht werden: Sie können berechenbare Zahlen innerhalb einer beliebigen Toleranz gemäß einer Verteilung generieren.
Ja absolut. Es ist eine tatsächliche Strategie, die man tatsächlich ausführen und mit einer Wahrscheinlichkeit von mehr als richtig vorhersagen kann 1 2 . Ich habe mich bemüht, dies in meinem ursprünglichen Beitrag zu erklären, einschließlich zum Beispiel „Methoden dafür sind gut untersucht“ und dergleichen, in der Hoffnung, die Zweifel zu überzeugen.
Ich denke, das ist bisher die beste Antwort in diesem Thread. +1 zu dieser Antwort und Ihrer vorherigen Antwort, die das Paradox erklärt hat.
Person A kann sehr leicht eine Strategie wählen, die die Chancen von Person B, richtig zu raten, sehr nahe bei 1/2 hält (nahe genug, dass es sich nicht lohnt, diese Strategie in der Praxis gegen einen sachkundigen Gegner anzuwenden), aber die Wahrscheinlichkeit ist größer immerhin 1/2. +1 für Sie.
Das ist der Fall. Ich habe eine solche Strategie von Person A in meinem ursprünglichen Beitrag beschrieben . Aber ich finde es auch interessant zu bemerken, dass A die Bindung nicht garantieren kann B 's Erfolgswahrscheinlichkeit zu ( 1 2 , 1 2 + ϵ ) für alle ϵ weniger als 1 2 .
@DanielV: "Die Verteilung, die Person A verwendet" ist ein Konzept, das für das vorliegende Problem fremd ist, und wenn ich das richtig verstehe, könnte die bloße Erwähnung dieses Satzes zu Verwirrung führen. Es gibt keine Verteilung ; Die Behauptung beinhaltet eine universelle Quantifizierung über alle Auswahlmöglichkeiten von A. Das heißt, wenn A 1 und 7 auswählt, funktioniert dies. Wenn A -sqrt(2) und 763,45 auswählt, funktioniert dies. Usw.
Ich denke nicht, dass es offensichtlich ist, dass es eine solche Strategie nicht gibt, ich denke sogar, dass die meisten Laien sie intuitiv anwenden würden (in einer heuristischen Form, dh "klingt diese Zahl groß?" im Sinne von "hätte ich wahrscheinlich gewählt zwei Zahlen und finden Sie diese als die größere"). Offensichtlich und in der Tat richtig ist, dass die Strategie von Person A leicht unterlaufen werden kann, indem beide Zahlen sehr groß und nahe beieinander gewählt werden.
Falls Sie es nicht glauben: jsfiddle.net/GeB8V oder Console thisfunction pickPos() { return Math.floor(Math.random() * Number.MAX_SAFE_INTEGER); }; function play() { var y = pickPos(); x = pickPos(); z = pickPos(); return ((x < y && x < z) || (x > y && x > z)) ? true : false }; counter = {wins: 0, losses:0}; i = 100000; while(i--) { counter[play() ? 'wins' : 'losses']++ }; counter
@qwertymk Dies setzt voraus, dass Person B die Wahrscheinlichkeitsverteilung von Person A kennt.
@qwertymk Es wird auch eine Obergrenze für die gewählten Zahlen angenommen.
@DanielV + Tyilo Abgesehen von diesen Punkten ist es immer noch ziemlich erstaunlich, dass es funktioniert
Das lässt sich ganz einfach erklären. B errät die Strategie von A. Wenn sie sich irrt, stehen ihre Chancen bei 1/2. Wenn sie Recht hat, stehen ihre Chancen besser als 1/2. Solange ihre Chancen, die Strategie von A zu erraten, besser als 0 sind, sind ihre Gewinnchancen besser als 1/2.
Das zusätzliche Element ist, dass, wenn wir dies in ein iteratives Spiel verwandeln, B ihre Vermutung auf eine Weise ändern muss, die A nicht mit 100%iger Sicherheit vorhersagen kann. Kurz gesagt, da B's Vorhersage von A immer eine Chance hat, richtig zu sein, und A's Vorhersage von B's Vorhersage immer eine Chance hat, falsch zu sein, ist B immer voraus.
@ AleksandrDubinsky Ich denke, du liegst falsch. Die eigentliche Methode von A besteht darin, aus N(0,1) zu wählen. B vermutet, dass die Methode von A darin besteht, aus N(3,1) zu wählen. B wird viel mehr als 50 % verlieren.
@MatthewFinlay In diesem Fall wird B fast immer sagen, dass sie den kleineren ausgewählt hat. Sie hat immer noch eine 50%ige Chance, tatsächlich den kleineren zu wählen.
@AD Wenn A ein Kontinuum von Strategien hat (z. B. Wahrscheinlichkeitsverteilungen), wie kann B mit der Wahrscheinlichkeit "es richtig erraten". > 0 ?
Ich habe mich vielleicht geirrt. Es gibt einige Vermutungen, die die Chancen von B ruinieren können. Beispielsweise besteht die Strategie von A darin, die hohe Zahl gerade und die niedrige Zahl ungerade zu machen. B vermutet, dass die Strategie von A umgekehrt ist und verliert immer. Wenn wir die Wahrscheinlichkeit mitteln, dass A hoch = gerade und hoch = ungerade entscheidet, liegt B wieder bei 50/50. Bei manchen Ratestrategien tut es weh, falsch zu liegen. Die von MJD vorgeschlagene Ratestrategie ist großartig, weil sie niemals zum Nachteil von B sein kann. Ich frage mich, ob es einzigartig ist oder ob es andere mit dieser Eigenschaft gibt.
Übrigens @MJD, danke fürs Posten; es ist eines meiner liebsten Wahrscheinlichkeitsprobleme!
@Goos Es gibt ein Kontinuum reeller Zahlen. Das hindert B nicht daran, den Median der Verteilung von A zu erraten (so interpretiere ich die offizielle Lösung). Wenn wir zB annehmen, dass die Verteilung von A irgendwo auf dem reellen Zahlenstrahl ein kleiner Bereich ist , hat B immer noch eine Chance ungleich Null, eine Zahl zu erraten, die darin liegt (und gewinnt).
@Goos Ok, ich verstehe. Der Trick des Beweises besteht darin, dass es zwischen zwei beliebigen reellen Zahlen eine rationale gibt. Es ist das Rationale, das mit einer Wahrscheinlichkeit ungleich Null erraten werden kann.
@Goos Bist du sicher? Eine rationale Zahl zu erraten, ist wie zwei ganze Zahlen zu erraten. Wenn ich keine rationale Zahl erraten kann, kann ich keine ganze Zahl erraten. Wenn ich eine ganze Zahl nicht erraten kann, kann ich keine ganze Zahl erraten, die in einen Bereich fällt. Wie kommt es, dass ich eine Realzahl im selben Bereich erraten kann? (Umgekehrt arbeiten: Wenn ich eine reelle Zahl zwischen zwei reellen Zahlen erraten kann, dann muss ich, um ein rationales m/n zu erraten, nur zwei reelle Zahlen in den Bereichen (m,m+1) und (n,n+1) erraten und abrunden .)
Bitte vermeiden Sie ausgedehnte Diskussionen in Kommentaren.
Es gibt eine Methode zum Generieren der Zahlen, für die nicht besser als 1/2 funktionieren kann.
@ Joshua Das ist falsch. Können Sie Ihre vermeintliche Methode erklären?
Indem wir die zitierte Metrik wieder einspeisen, können wir trivialerweise ein Auswahlmuster erzeugen, das diese Metrik durchbricht. Natürlich gibt es ein Gegenmuster, um diesen zu besiegen. Die zufällige Auswahl zwischen L1- und L2-Strategien sollte 1/2 ergeben.
@ Joshua Du irrst dich. Selbst wenn X weiß, dass Y Kommissionierung von einer Normalverteilung ist, gibt es nichts X tun können, um zu reduzieren Y 's Erfolgswahrscheinlichkeit zu 1 2 .
Das Werfen einer Münze ist ein Ablenkungsmanöver; Wenn ich eine Strategie will, bei der meine Gewinnchance 1/2 beträgt, dann kann ich einfach immer die Karte behalten, die ich zuerst wähle (oder immer wechseln). Um es besser zu machen, muss ich mich nur dazu verleiten lassen, lieber eine größere Anzahl zu behalten. Die einfachste verbesserte Strategie besteht darin, eine positive Zahl beizubehalten und zu wechseln, wenn ich eine negative Zahl erhalte (obwohl ich stattdessen genauso effektiv mit einer anderen Zahl als 0 vergleichen könnte). …
… Wie gut das funktioniert, hängt davon ab, wie A die Zahlen wählt. Wenn A immer zwei positive Zahlen wählt, dann ist dies gleichbedeutend damit, dass ich immer meine Zahl behalte, also keine Verbesserung. Ähnlich, wenn A immer zwei negative Zahlen auswählt. Aber wenn A immer eine positive Zahl und eine negative Zahl wählt, dann ist diese Strategie perfekt und ich gewinne immer. Um das Problem zu lösen, muss ich meine Wahrscheinlichkeit über 1/2 erhöhen, egal was A tut, also ist dies noch keine Lösung, aber es sollte ausreichen, um die Idee zu verbannen, dass keine Verbesserung möglich ist. …
… Um eine vollständige Lösung zu erhalten, muss ich nur meinen Cut-off-Punkt zufällig auswählen. Da A die Zahlen auf einen beliebigen begrenzten Bereich beschränken kann, möglicherweise ziemlich groß (oder klein) und eng, brauche ich eine Wahrscheinlichkeitsverteilung, die jedem nichttrivialen Intervall von Zahlen eine positive Wahrscheinlichkeit zuweist. Aber es gibt viele solcher Distributionen, also wählen Sie eine aus, verwenden Sie sie, um zufällig einen Cut-Off-Punkt auszuwählen, und wechseln Sie, wenn meine Zahl unter dem Cut-Off-Punkt liegt. Und das ist die vom OP beschriebene Lösung.

Dies ist im Vergleich zu den meisten anderen Beispielen elementar, aber wie wäre es

Es gibt mehr rationale Zahlen als ganze Zahlen.

Die Schönheit liegt in seiner Einfachheit. Die Falschheit dieser einen dieser sehr elementaren Beobachtungen weigert sich der Teil meines Gehirns, der vom gesunden Menschenverstand getrieben wird, immer noch zu akzeptieren.
Mein Gehirn weigert sich im Allgemeinen, dies zu akzeptieren. :) Anscheinend auch wenn man sich einschränkt B Zu { 1 , A + 1 } , A / B gibt Ihnen doppelt so viele Werte wie A .
Und es gibt halb so viele ungerade ganze Zahlen wie (ungerade oder gerade) ganze Zahlen.
Das wusste ich nicht, könnten Sie erklären, was an dem folgenden (falschen) Beweis falsch ist. ( N N ) ( M 1 = N , M 2 = N + 1 2 ) R , was bedeuten sollte, dass es zu jeder ganzen Zahl mindestens zwei reelle Zahlen gibt. Ich denke, das Problem liegt in der Unendlichkeit der natürlichen Zahlen, und es ist ein bisschen so, als würde man sagen 2 > , was offensichtlich falsch ist. Ist das der Haken?
@Jori: Ich habe die Beweise und die Terminologie seit Jahren nicht mehr studiert, aber ich denke, eine etwas einfache (vereinfachte?) Art, es auszudrücken, ist, dass die Existenz einer Injektionsabbildung (nicht eins zu eins) die Existenz nicht ausschließt einer Eins-zu-Eins-Abbildung. PS Einige Leute halten 2∞>∞ vielleicht für offensichtlich wahr .
@Jori: Der Satz besagt rationale Zahlen, keine reellen Zahlen. Und der klassische Beweis ist, afaik, (Cantor's diagonalization)[ en.wikipedia.org/wiki/Cantor's_diagonal_argument] . Sie können eine Strategie entwickeln, die die rationalen Zahlen deterministisch aufzählt. Dies ordnet jeder rationalen Zahl eine natürliche Zahl zu, was zeigt, dass die Mengen die gleiche Größe haben.
Ups, falsch gelesen. Werde mal in deinen Link schauen. Danke! Gilt das auch für reelle Zahlen?
@Jori: (1) Ich hätte sagen sollen: „Die Existenz einer nicht-surjektiven Injektionsabbildung (keine Eins-zu-Eins-Korrespondenz) zwischen zwei transfiniten (dh unendlichen) Mengen schließt die Existenz einer Eins-zu- eine Korrespondenz. Offensichtlich schließt die Existenz einer nicht-surjektiven Injektionsabbildung zwischen zwei endlichen Mengen die Existenz einer Eins-zu-Eins-Korrespondenz aus. (Oder ist dies eine weitere „offensichtliche“ Wahrheit, die nicht wirklich wahr ist?)
@Jori: (2) Nein! Wie Cornelius in seinem Kommentar angedeutet hat, zeigt Cantors diagonales Argument (korrekter Link), dass es unendliche Mengen gibt (wie die Menge der reellen Zahlen), die nicht in eine Eins-zu-Eins-Korrespondenz mit der (unendlichen) Menge der natürlichen Zahlen gebracht werden können . Es gibt (unendlich viele!) verschiedene transfinite Kardinalzahlen . Grob gesagt sind nicht alle Unendlichkeiten gleich.
@ gnasher729: Sicherlich ist es weniger falsch als bedeutungslos oder wahr zu sagen, dass es halb so viele ungerade ganze Zahlen gibt wie ganze Zahlen. Das Überraschende ist das 2 0 = 0 .

Ich rede immer wieder darauf herum, weil ich denke, dass es ein spektakuläres Beispiel für etwas ist, das sich als völlig offensichtlich erweisen lässt (nicht nur, weil es so scheint, sondern weil es so lange so weit verbreitet war) und doch völlig falsch ist:

Vermuten Φ ist eine Eigenschaft, die für ein Objekt gelten kann oder nicht. Dann gibt es eine Sammlung S Φ aller Objekte mit Eigentum Φ .

Viele ernsthafte und sogar berühmte Mathematiker gingen mit diesem intuitiv offensichtlichen, aber völlig falschen Prinzip vor, dessen Zerstörung die Mathematik in ihren Grundfesten erschütterte und den Beginn der modernen Logik und Mengenlehre markiert.

(Es gibt viele Gegenbeispiele, von denen das bekannteste ist Φ ( X ) = X ist kein Mitglied der Sammlung X “. Für andere siehe Ist das Russell-Paradoxon der einzig mögliche Widerspruch zum Axiomschema des Verständnisses von Frege (1893)? { X : P ( X ) } und Paradox des allgemeinen Verständnisses in der Mengenlehre, außer Russells Paradox .)

+1 für ein großartiges Beispiel, aber ich denke, die Verwendung von "Eigenschaften", "Objekten" und "Sammlungen" beeinträchtigt die Wirkung, indem sie ungenau ist. Es gibt definitiv konsistente Mengentheorien, bei denen es Sammlungen gibt, die keine Objekte sind, und die Aussage in gewissem Sinne wahr ist. Die Angabe mit den Wortmengen und im Kontext der traditionellen naiven Mengenlehre und der modernen weit verbreiteten Formalisierungen der Mengenlehre würde diese Antwort meiner Meinung nach besser machen.
Warum ist das falsch?
Ich bin anderer Meinung, ich denke, das funktioniert ganz gut mit undefinierten Begriffen, solange Sie eine Vorstellung von Containment haben, brauchen Sie nichts anderes.
Josua hat hier Recht, und dieses widersprüchliche Ergebnis trifft auf sehr viele verwandte Situationen zu. Zum Beispiel sagt WV Quine: „Es ist nicht zu leugnen, dass dieses Prinzip stillschweigend ständig verwendet wird, wenn wir von Adjektiven sprechen, die für Dinge wahr sind: Das Adjektiv ‚rot' ist wahr für ein Ding, wenn und nur wenn das Ding rot ist, und entsprechend für alle Adjektive … Es ist ein hartes Prinzip, dem man misstrauen kann, und doch … [es] muss aufgegeben oder zumindest irgendwie eingeschränkt werden.“ Ebenso könnte man denken, dass es möglich wäre, einen Katalog zu erstellen C Φ aller Bücher mit etwas Eigentum Φ , aber es ist nicht.
@mattecapu Es gibt keine Sammlung S Φ aller Sammlungen, die nicht selbst Mitglieder sind.
Oh, ich verstehe deinen Punkt
Ich habe begonnen, diese Formulierung des Problems irreführend zu finden. Man kann immer eine "Sammlung" von Objekten betrachten, die eine Formel erfüllen, da man sie individuell und begrifflich betrachten kann, diejenigen, die dies tun, werden von denen getrennt, die dies nicht tun. Das Problem ist genau, was ein "Objekt" ist und welche Sammlungen davon eine "Menge" bilden. In ZF sind sie dasselbe, aber Sammlungen, die durch willkürliche Formeln definiert sind, sind es möglicherweise nicht; das ist etwas anderes als zu sagen, dass sie nicht existieren: Sie unterliegen vielmehr nicht den Axiomen als "Objekte".
Ich denke wirklich, dass genau das nicht das Problem ist. Das Problem hier hat nichts mit ZF, mit Axiomen oder damit zu tun, wie wir „Objekte“ genau definieren; Es ist ein grundlegendes Problem mit der intuitiven Vorstellung davon, was es bedeutet, wenn Dinge Eigenschaften haben. Was halten Sie von dieser Formulierung: „Für jede Immobilie Ψ , kann man einen Katalog erstellen, der alle Bücher mit Eigentum auflistet Ψ ?” Und die Antwort ist, dass für einige Eigenschaften Ψ , du kannst es einfach nicht. Ist das hinreichend konkret und nicht mengentheoretisch?
Wow, danke, ich habe von Bertrands Paradoxon gehört, aber ich habe es immer als ein erfundenes Beispiel betrachtet, das einen Fehler im aktuellen Denken der Zeit demonstriert. Es als Gegenbeispiel dazu zu sehen, zeigt wirklich, wie verängstigt die mathematische Gemeinschaft gewesen sein muss.
Ich neige dazu zu glauben, dass es kein Problem ist, einen Katalog roter Dinge zu haben, da die Definition nicht rekursiv ist. Können Sie einen Hinweis auf eine Diskussion geben, die zeigt, wann diese Intuition gefährlich ist?
@Polymer Das ist in der Tat eine Möglichkeit, wie Leute versucht haben, dieses Problem zu lösen - das Zauberwort für diese Art von Einschränkung (dass sich jeder Satz nur auf eine „untere Klasse“ von Sätzen bezieht) ist Typentheorie . Der Haken ist dann, dass es viele mathematische Konstruktionen gibt, die sehr schwer 'typsicher' zu machen sind.
Nein, diese Definition ist zirkulär.
@MJD Egal wie Sie es mit umgangssprachlichen Begriffen sagen, Sie führen mich implizit dazu, eine "Sammlung" als eine Art "Objekt" oder einen "Katalog" als eine Art "Buch" zu betrachten. Das ist weder notwendig noch offensichtlich; Tatsächlich ist die Behauptung, dass es wahr ist , genau der Inhalt des Axioms der Spezifikation (unter den entsprechenden Umständen). Theoretisch können die Objekte, die die Formel erfüllen, als Ganzes betrachtet werden, aber dieses Ganze ist nicht notwendigerweise ein Objekt der Art, wie sie von den Axiomen beschrieben wird. (IOW: nein, ein Katalog ist kein Buch, daher überzeugt mich Ihre Umformulierung nicht.)
„dessen Zerstörung die Mathematik in ihren Grundfesten erschütterte und den Beginn der modernen Logik und Mengenlehre markiert“ Wo in der Geschichte der Mengenlehre kommt das ins Spiel?
Dies ist Russells Paradox , das 1901 entdeckt wurde und Freges Grundgesetze der Arithmetik (1903) und frühere Arbeiten zerstörte. Zu den unmittelbaren Reaktionen auf das Paradox gehören Whitehead und Russells Principia Mathematica (1910) und Zermelos Arbeit zur axiomatischen Mengenlehre ab 1905, die schließlich zu ZFC wurde, das die Mengenlehre bis heute dominiert.
Erfordert das Paradoxon, dass eine Sammlung als Objekt gelten muss? Was ist, wenn die Eigenschaft einfach „das Buch befindet sich nicht in einem Katalog“ lautet? Sie können keinen Katalog von nicht katalogisierten Büchern erstellen. Einfach ausgedrückt, wenn Sie Definitionen (von Eigenschaften oder was nicht) so ausgefeilt lassen, dass sie Turing-vollständig (oder einfach kreisförmig) sind, können Sie nicht erwarten, dass sie immer anhalten oder sogar wissen, ob sie anhalten werden .
Wäre das logische Äquivalent "Diese Aussage ist falsch"?
Nein, das ist überhaupt nicht gleichwertig. Warum denkst du, dass es so wäre?
@Jab das "logische Äquivalent" ist
definieren  P ( Q ) := ¬ Q ( Q )
An der Oberfläche sieht es nicht rekursiv aus, aber was ist mit P (P) ...
@MJD Vom Fehlerfall, meinte ich.
Ich sehe nicht, was der Unterschied ist. Die naive Mengentheorie ging davon aus, dass jede definierbare Menge existieren muss. Es ist, als würde man sagen, dass jede konstruierbare Aussage entweder wahr oder falsch sein muss. „Diese Aussage ist falsch“ ist weder wahr noch falsch.
Ich glaubte, eine solche Aussage in Principia Mathematica von Russel & Whitehead gesehen zu haben , und nahm an, dass ihr Typensystem sicherstellte, dass es keine Widersprüche gab.
Ich denke, "Diese Aussage ist falsch" hat einen unfairen Bekanntheitsgrad im Vergleich zu "Diese Aussage ist wahr", was in der realen Welt viel mehr Ärger verursacht.
@R..: Ich stimme nicht zu; Die Verwendung einer nicht-mathematischen Sprache ist erforderlich, um den Durchschnittsmenschen zu überraschen, dass sie falsch ist.

In einem verwandten mathOverflow-Thread wies Gowers auf die folgende offensichtliche, aber falsche Behauptung hin:

Lassen ICH 1 , ICH 2 , Teilintervalle von sein [ 0 , 1 ] deren Gesamtlänge strikt kleiner als 1 ist. Dann ist die Vereinigung der ICH ich kann nicht enthalten Q [ 0 , 1 ] .

(Beachten Sie, dass wenn Q wird durch ersetzt R , die Behauptung ist wahr.)

Ich finde die Tatsache, dass alle Q kann von einer willkürlich kleinen Familie von Intervallen abgedeckt werden, um eine der bizarrsten kontraintuitiven in der gesamten Mathematik zu sein.

Interessant! Gibt es ein explizites Beispiel für eine solche Vereinigung oder kann sie nur nachgewiesen werden?
Sie bestellen die Begründungen Q 1 , Q 2 , , Q N , und platzieren Sie ein Intervall von ( ϵ 2 ) N um jeden herum, wählend ϵ kleiner sein als 1 .
Natürlich gilt der Satz auch dann noch, wenn wir eine endliche Anzahl von Intervallen haben; man braucht die unendliche Konstruktion, um diese Arbeit zu machen, was wahrscheinlich dort liegt, wo die Lücke in der Intuition liegt.
@dfan funktioniert das für jedes e, das kleiner als 1 ist? Und mit Ordnen der Rationalen meinst du das Ordnen der Rationalen, die in [0, 1] sind? Die Konstruktion fühlt sich für mich immer noch schlecht an. Sie haben es so formuliert, dass die Reihenfolge der Rationen willkürlich ist, aber wenn die erste rationale Zahl 0,00000001 für e = 0,9 war, dann wird das Intervall dafür weit außerhalb von [0,1] liegen. Was fehlt mir hier?
Nebenbei führt das obige Nicht-Theorem ein wichtiges Konzept zum Verständnis der Integration ein. Verknüpfung
@Cruncher Ja, es funktioniert für alle ϵ < 1 seit ich = 1 ( ϵ 2 ) N < 1 . Und ja, er bezieht sich auf die Begründungen in [ 0 , 1 ] Wenn Sie jedoch über das Argument nachdenken, werden Sie feststellen, dass dies nicht unbedingt erforderlich ist, er könnte alle rationalen Argumente mit endlicher Länge abdecken. Schließlich war die Aussage nicht, dass die Intervalle gleich wären Q [ 0 , 1 ] , aber dass sie es enthalten.
@cruncher Sie müssen zuerst die Begründungen in eine Liste aufnehmen, sagen wir 1 1 ; 1 2 , 2 1 ; 1 3 , 2 2 , 3 1 ; 1 4 , 2 3 , 3 2 , 4 1 ; . Dann um die N th rational in der Liste, setzen Sie ein Intervall von width ϵ 2 N . Das heißt, wenn die N rational ist Q , dann ist die N Intervall ist ( Q ϵ 2 ( N + 1 ) , Q + ϵ 2 ( N + 1 ) ) , was sicherlich beinhaltet Q , und die Länge hat ϵ 2 N . Und Sie können haben ϵ sei so klein wie du willst. Und doch ist jedes Rationale in einem dieser Intervalle enthalten.
Tut [ A , A ] als Intervall zählen? Dann ist die ICH ich kann nur eine Ordnung der rationalen Zahlen sein [ 0 , 1 ] .
@alex.jordan: nein - die Anforderung sind normalerweise entweder offene Intervalle oder Intervalle mit positivem Maß.
@Cruncher Du hast Recht, das ist ein Problem, weil in der Antwort "Subitintervals" steht. Also zwei Dinge: 1) Ich glaube, dass die Konstruktion so festgelegt werden kann, dass unsere Intervalle nach außen gehen [ 0 , 1 ] ; zum Beispiel (dies muss jedoch ein wenig überprüft werden, das könnte unmöglich sein), wenn einige Ihrer Teilintervalle außerhalb liegen [ 0 , 1 ] , einfach weiter schrumpfen ϵ bis jedes Subintervall drin ist [ 0 , 1 ] . 2) Auch wenn Sie das nicht können, ist es immer noch erstaunlich, dass Sie die Rationalen mit Intervallen überdecken können, deren Summe beliebig klein ist, ob Sie alle Teilintervalle hineinpassen [ 0 , 1 ] oder nicht.
@Cruncher Es sagt uns, dass die rationalen Argumente wirklich vernachlässigbar sind, wenn Sie versuchen, die Länge zu messen. Sie können beispielsweise sagen, dass die Länge von [ 0 , 1 ] Q = 1 . Wenn Sie mehr interessiert sind, müssen Sie nach "Lebesgue-Maß" googeln. Die Rationalen haben das Lebesgue-Maß 0 , und dies ist die Standardmethode, um es anzuzeigen.
@Cruncher Numberphile hat ein interessantes Video auf youtube.com/watch?v=aIggWlKr41w
@Polymer: Richtig, das Konzept einer Menge von Maß Null.
@Ovi: Die Teilintervalle müssen nicht auf die Rationalen zentriert sein, also musst du ε nicht verkleinern, nimm einfach den Schnittpunkt mit [0,1].
Dies ist keine beliebig kleine Familie von Intervallen, sondern eine Familie von Intervallen mit beliebig kleinem Gesamtmaß.
Dieses Problem wurde in einem 3Blue1Brown-Video über Musiktheorie diskutiert

Wenn eine Funktion F ( X ) hat also eine horizontale Asymptote lim F ' ( X ) = 0

+1 Ich liebe dieses! Ein Gegenbeispiel ist Sünde X 2 X .
Eine hinreichende Bedingung dafür ist, dass die zweite Ableitung beschränkt ist.
@AP oder auch nur das lim F ' ( X ) existiert.
Als Student versuchte ich, eine zu schreiben ε - δ Beweis, dass wenn F ' ( A ) > 0 dann gibt es ein offenes Intervall, das enthält A auf welche F nimmt zu. Und ich dachte an meine Fähigkeit zu schreiben ε - δ Proofs fehlte etwas, weil ich nicht herausfinden konnte, wie es geht.
Ah, wir haben fast doppelte Antworten. Das Interessante ist, dass Sie die folgenden Einschränkungen hinzufügen können, um es noch bizarrer zu machen: F Ist C , beschränkt und monoton.
@Kaj Hansen: Kannst du mir ein Beispiel zeigen F ? :)
Obwohl ich glaube, ich kann es mir eigentlich vorstellen: denk an einen F Das hat glatte, aber immer steilere und immer kürzere Treppenstufen, wenn es grafisch dargestellt wird. Dann ist die lim ist nicht vorhanden. (Warum?) Aber was ist mit einem, wo die lim existiert?
Ich denke, was Sie beschreiben, kann tatsächlich mit etwas Basteln funktionieren. Siehe meine 10-Upvote-Antwort unten.
@mike4ty4 Wenn das Limit existiert, muss es gleich 0 sein.
Stimmt diese Aussage noch, wenn F ( X ) darf seine eigene horizontale Asymptote nicht kreuzen? Weil ich denke, dass die falsche Intuition daher kommen könnte.
@Ryan Du meinst wenn F ( X ) ist monoton? Ja, auch in diesem Fall können wir nicht sagen, dass die Grenze gleich ist 0 , wie andere darauf hingewiesen haben
Eine unter Analphabeten weit verbreitete Definition ist, dass eine Asymptote eine Linie ist, der sich der Graph einer Funktion immer nähert, aber nie berührt.
@Ryan Wenn du nimmst F ( X ) = Sünde ( X 2 ) + 2 X , Dann F ( X ) ist schließlich positiv, aber die Ableitung hat immer noch keine Grenze.
@MichaelHardy bezüglich „Analphabeten“: Widersprechen Sie dem Teil „berührt nie“ oder der Unbestimmtheit der Definition? Wenn ersteres, dann ist das ein bisschen hart, wenn man bedenkt, dass dies buchstäblich (ha!) Die Etymologie des Wortes ist.
In erster Linie der Teil "nie berührt": Sie werden Ihnen sagen, dass, wenn eine Kurve eine Linie kreuzt oder berührt, diese Linie per Definition keine Asymptote der Kurve ist. Aber auch die Funktion X j = Sünde X X hat j = 0 als Asymptote, aber es kehrt nicht nur immer wieder zurück, um diese Linie zu überschreiten, sondern kommt auch nicht immer näher, nicht einmal "irgendwann".
Leider habe ich diese "Tatsache" ständig benutzt, um Gleichgewichtslösungen für Differentialgleichungen zu finden. Jetzt ist meine ganze Arbeit in Gefahr.

0. 9 ¯ < 1

Wahrscheinlich das berühmteste der "offensichtlichen", aber falschen.

Ich frage mich, wie "offensichtlich" das bei Leuten ist, die wissen, wie die reellen Zahlen tatsächlich definiert sind.
Die Frage stellte, was die durchschnittliche Person für wahr halten würde.
Eine andere Möglichkeit, dies auszudrücken, ist "jede reelle Zahl kann auf genau eine Weise als unendlich lange Dezimalerweiterung geschrieben werden", um es für die meisten Menschen offensichtlicher zu machen.
Dies ist bei weitem das beste auf der Seite imo. Einige der anderen sind für den Durchschnittsmenschen so komplex, dass sie für fast niemanden „offensichtlich“ wahr wären.
In reellen Zahlen ist es falsch, aber es ist wahr für die Hyperrealen. Es ist nur eine Konvention, dass wir standardmäßig die Theorie der reellen Zahlen anstelle eines anderen Systems verwenden.
Will keine Art von Zahlen berücksichtigen 0,999 ... geradeaus ist eine Zahl, aber dieses "Offensichtliche" ist tatsächlich wahr, lassen Sie mich erklären. 0,999... ist gleich 1 - 1/∞. Was eine seltsame Art von Unendlich ergibt, die unendlich nahe an Eins liegt.
Aber 0,999... ist nicht gleich 1 - 1/∞ (es sei denn, Sie meinen mit 1 - 1/∞ nur 0).
Ich habe das geglaubt, bis ich es war 9. 9 ¯ Jahre alt. Auf meinem 10 th Geburtstag, ich sah das Licht. :-) ⁠
Denken Sie daran, dass eine unendliche Kette von 0 s endet mit a 1 endet nie in a 1 , denn das würde der Idee widersprechen, dass die Unendlichkeit niemals endet

Die Falschheit von

Lassen S sei eine unendliche Familie streng positiver Zahlen. Dann S =

verblüfft die Menschen seit Tausenden von Jahren. Es ist die Grundlage für Zenos Paradoxon, aber wenn Sie denken, dass Zenos Paradoxon alt und müde ist, bedenken Sie, dass es auch die Grundlage für das Gabriel's Horn-Paradoxon (auch in diesem Thread erwähnt) ist, das die Leute immer noch verwirrt .

Das ist meiner Meinung nach das beste Beispiel hier. Es ist für einen Laien mit einer kleinen Erklärung leicht verständlich, und auf den ersten Blick scheint es unglaublich offensichtlich, dass es immer größer werden würde, wenn man die Zahlen weiter addiert.
@ jpmc26 Es wird immer größer, wenn Sie weiter Zahlen hinzufügen. es tut dies nur nicht unbedingt unbegrenzt.
Wenn andererseits die Familie nicht zählbar ist, gilt sie.
Ich habe gerade dieses und die Kommentare gelesen ... und es ist wirklich freakig. (Ich habe einen Master in Physik, also bin ich mit solchen Dingen nicht ganz unbekannt.)
Ich habe das Gefühl, dass das nicht so verrückt ist, obwohl das vielleicht daran liegt, dass ich Erfahrung habe, aber ich meine 1.1111 = 1 + 0,1 + 0,01 + .
@Almo Sie haben diese höchstwahrscheinlich in Ihren Grundkursen in Analysis gelernt. Es ist nur eine unendliche Summe, und einige unendliche Summen konvergieren zu einem Wert, anstatt zu divergieren. Erinnern Sie sich an all diese Reihen-Konvergenz-/Divergenz-Tests zu der Zeit, als Sie zum ersten Mal Integrale studierten?
Ooooh! Ich lese es als "streng positive ganze Zahlen ", von denen ich glaube, dass sie divergieren würden. Rechts? (Zu viel Programmierung)
@Almo Die Summe einer unendlichen Reihe positiver Ganzzahlen divergiert notwendigerweise gegen unendlich, ja (sie kann nicht kleiner sein als die Summe einer unendlichen Reihe von Einsen).
@ user87690 Was wäre wenn S S , X R + 0 : S < X ; das ist, S ist unendlich klein?
@WChargin: Dann gilt es wohl nicht mehr, da die endliche Summe von Infinitesimalen infinitesimal ist, also ist jede endliche Untersumme kleiner als sagen 1 . Für meinen vorherigen Kommentar bin ich davon ausgegangen, dass eine streng positive Zahl eine reelle Zahl ist.
@ user87690 Ich denke nicht, dass das richtig ist. Die Summe einer unzählbaren Anzahl positiver Infinitesimale wird nach dem Übertragungsprinzip nicht konvergieren (da sie für keine positive reelle Zahl konvergiert). Alternativ konvergiert es nicht, weil es schließlich mehr als ergeben wird 1 nachdem Sie eine hyperganzzahlige Anzahl von Termen erreicht haben (immer noch weniger als ω ).
@MarioCarneiro Ich sage nicht, dass es konvergiert, ich sage nur, dass es nicht ∞ ist. Um klar zu sein, die Summe, von der ich spreche, ist als Grenze des Netzes endlicher Teilsummen definiert. Ist es nicht wahr, dass jede endliche Summe von Infinitesimalen nach gewöhnlicher Induktion infinitesimal ist?
@ user87690 Ich kann mich irren, aber ich glaube, das ist falsch, in dem Sinne, dass Sie, wenn Sie versuchen aufzuschreiben, was Sie tatsächlich mit "endlich" meinen, entweder explizit auf nicht standardmäßige Zahlen verweisen oder sonst gezwungen sind, nicht standardmäßige natürliche Zahlen zuzulassen auch "endlich" sein. Betrachten Sie der Einfachheit halber stattdessen eine zählbare Summe gleicher Zahlen (die nicht zählbare Summe reduziert sich auf diesen Fall). Für jede positive Zahl ϵ es gibt eine Reihe von Begriffen N so dass die Summe größer als ist 1 (und durch Multiplikation mit M Sie können es so groß machen, wie Sie möchten). (Fortsetzung)
... Durch das Übertragungsprinzip gilt dies auch für Infinitesimal ϵ , obwohl jetzt N ist auch eine nicht standardmäßige natürliche Zahl (aber immer noch "endlich"). Mit anderen Worten, wenn Sie sich selbst millionenfach ein Gazillionstel hinzufügen, erhalten Sie immer noch eins. Wenn Sie die Existenz unendlich kleiner Zahlen akzeptieren, sind Sie gezwungen, auch Hyperzahlen zu akzeptieren, und so divergiert die Summe immer noch zu .
Die Aussage, die Sie zu machen versuchen, ist, dass eine Summe von N unendlich kleine Zahlen wo N Dieser Standard ist ebenfalls infinitesimal, aber dies kann innerhalb des Modells nicht angegeben werden. (Alternativ könnte man sagen, dass die Summe von 2 und 3 und 4 Infinitesimalen Infinitesimal ist, aber nicht allgemein N , da es auch für jedes Infinitesimal gilt ϵ Da ist ein N so dass N ϵ > 1 .)
@ user87690 Es gibt endliche Integrale 1 j , j > 0 Ich würde also erwarten, dass eine unzählige Anzahl positiver Werte immer noch eine endliche Summe haben könnte.
@MarkHurd S N := { X S : X > 1 / N } . Wenn einige S N ist unendlich, dann klar S = . Andererseits, wenn sie alle endlich sind, dann S da ihre Vereinigung zählbar ist.
@MarioCarneiro Ich verstehe, du hast Recht. Aber ich habe nicht versucht, irgendetwas innerhalb des Modells anzugeben. Wir haben nur Objekte ω , R mit elementarer Einbettung zu einigen R . Infinitesimals sind bestimmte Mitglieder von R , wir fassen zusammen R und die unendliche Summe ist eine Grenze des Netzes endlicher Untersummen, wobei endlich bedeutet, dass ein Mitglied bijektiv ist ω .
Zeno ist mit 9 überlebenden bis 40 zugeschriebenen Argumenten verbunden. Alle Referenzen. Es ist fehleranfällig, ihn mit einer Variante dessen in Verbindung zu bringen, was diese Antwort besagt. Jeder, der daran interessiert ist, eine solche behauptete Assoziation zu validieren, kann den Artikel auf mathpages.com / Kevin Brown's Reflections on Relativity aufschlussreich finden. Es versucht, darzustellen, was tatsächlich über Zenos Arbeit bekannt ist, und zu zeigen, dass die beiden berühmtesten Paradoxien „ weit davon entfernt, von der Konvergenz unendlicher Reihen beeinträchtigt zu werden, sondern tatsächlich von der Konvergenz der geometrischen Reihen abhängen “.
@ user87690 Akzeptiert, und der Fehler bei meiner Aussage sind die Level-Sets (was deine S N sind) eines Integrals sind alle unendlich, aber alle Infinitesimale.
Das ist interessant S = Und 1 / N = C ; F ich N ich T e sind beide auf dieser Liste. Wie wäre es also mit "wenn etwas offensichtlich wahr ist, kann es nicht offensichtlich falsch sein" als P Ö S ich T ich v e = ist eindeutig beides.

Jede Kette von Teilmengen von N ist zählbar.

Bisher finde ich das für mich am zutreffendsten. Für mein Leben kann ich mir immer noch keine gute Metapher geben, die es offensichtlich falsch macht, obwohl ich mathematische Gegenbeispiele kenne. Andere Antworten, entweder hat es sie nie offensichtlich gefunden, oder wenn ich es getan habe, kann ich das Missverständnis schnell durch eine alternative Betrachtungsweise beheben. Sie sollten wahrscheinlich ein Gegenbeispiel für dieses in der Antwort hinzufügen.
Meinen wir mit "Kette" eine Sammlung von Teilmengen von N, die durch die Teilmengenbeziehung vollständig geordnet ist? Sie können das Intervall darstellen [ 0 , 1 ] als solche Kette. Für jede Zahl X in diesem Intervall enthält die Teilmenge (abgerundet) die erste 9 X einstellige Zahlen, die erste 90 X zweistellige Zahlen, die erste 900 X dreistellige Zahlen usw.
@TannerSwett Das ist die Bedeutung, und deine ist ein gutes Beispiel.
@TannerSwett Betrachten Sie einfacher Dedekind-Schnitte rationaler Zahlen.
@DustanLevenstein Sicher. Oder lassen Sie Z [ ich ] sei die Menge aller Gaußschen ganzen Zahlen und sei S a = { z Z [ ich ] : 0 < A R G ( z ) < a } .
Der Beweis erschien in einigen Beiträgen auf dieser Seite, zum Beispiel hier und hier .

Kellers Vermutung ist offensichtlich wahr:

Lassen R N vollständig mit identischen, nicht überlappenden bedeckt sein N -Würfel. Es müssen zwei Würfel sein, die eine Seite teilen.

(Zum Beispiel wann N = 2 Wir bedecken die Ebene mit kleinen quadratischen Kacheln, und die Vermutung besagt, dass es zwei Kacheln geben muss, die eine Kante teilen. Das ist wahr.)

Die Vermutung ist jedoch für alle falsch N > 7 .

Ich bin mir nicht sicher, ob ich dieses Beispiel selbst kaufe, weil ich nicht sicher bin, ob irgendetwas in Bezug auf 7-dimensionale Dinge als intuitiv offensichtlich angesehen werden kann.
Wäre nicht die Tatsache, dass es wahr ist für N = 1 , 2 , 3 (Und N = 4 , 5 , 6 ) lassen Sie vermuten, dass dies für alle gelten könnte N ?
Was bedeutet „Gesicht“ überhaupt in 7 Dimensionen? Ist es ein 6-dimensionales Ding oder ein 2-dimensionales Ding?
Was genau meinen wir in diesem Zusammenhang mit „ein Gesicht teilen“?
Es ist 6-dimensional: der Hyperwürfel ist [ 0 , 1 ] 7 und Sie erhalten ein Gesicht, indem Sie eine Koordinate festlegen 0 oder 1 .
was meinst du mit "fliesen"?
Bitte geben Sie eine Referenz für diese Tatsache an.
Ohne weitere Qualifikation würde ich davon ausgehen, dass Kacheln auf einem n-Gitter gemeint sind, was meiner Meinung nach bedeutet, dass die Vermutung trivialerweise wahr ist. Vielleicht ist das offensichtlich nicht das, was Geometer gemeint ist, aber ich denke, dass andere Nicht-Geometer wie ich den gleichen Fehler machen könnten. Ich denke, es wäre hilfreich zu erwähnen, was wirklich mit einer "Fliese" gemeint ist.
Warum denkst du das? Das ist im Allgemeinen nicht gemeint, wo es alle möglichen Kacheln gibt, die keine quadratischen Gitter sind. Kacheln bedeutet in der Mathematik einfach, dass Sie die Ebene (oder was auch immer) mit den Kacheln so bedecken, dass sie sich nicht überlappen, außer an ihren Grenzen.
@ Jim zum Beispiel R 3 konnten von Flugzeugen in einem regelmäßigen Muster bedeckt werden, die gegeneinander gedreht waren. Ich kann mir gar nicht vorstellen, wie das funktionieren soll R 8 .
@gnasher729: Ich meinte nur, dass die in der Aussage verwendete Sprache verwirrend war, aber MJD hat ihre Antwort jetzt bearbeitet, damit sie klarer ist.
Was ist mit höherdimensionalen "Rändern"? (zB für N = 8 , es gibt keine geteilt ( N 1 ) -Würfel, aber was ist ( N 2 ) -Würfel und so weiter?) Oder gilt das Scheitern der Vermutung für N > 7 bedeuten, dass Kanten auch nicht geteilt werden müssen?
@gnasher729 Ich muss anfangen, in höheren Realitätsebenen zu leben.
Dieses Thema könnte einen netten Beitrag dazu leisten, wie versagt Intuition für höhere Dimensionen? sowie.
  • Wenn U ist eine offene Teilmenge von R N das ist homöomorph zu R N , man könnte es für "offensichtlich" halten, dass es tatsächlich diffeomorph zu ist R N (vielleicht so etwas denken wie "topologisch sieht es so aus R N , und differenzierbar ist es lokal trivial"). Tatsächlich gilt dies (aber keineswegs offensichtlich!) für N 4 . Aber für N = 4 es ist falsch: es gibt Exoten R 4 's (differenzierbare Mannigfaltigkeiten, die homöomorph, aber nicht diffeomorph sind, bis R 4 ), einschließlich "kleiner", die zu einer offenen Teilmenge von diffeomorph sind R 4 .

  • Viel weniger tiefgründig, aber immer noch lustig: Es ist "offensichtlich", dass die Summe zweier konvexer offener Mengen in der Ebene deren Rand ist C hat auch eine C Grenze (vielleicht denken Sie so etwas wie "Die Grenze der Summe wird durch eine glatte Funktion der Grenzen der Summanden parametrisiert"). Aber das ist falsch: Tatsächlich ist die Grenze der Summe immer C 20 / 3 (d. h. sechsmal differenzierbar und mit einer sechsten Ableitung, die passenderweise Hölder ist) und nicht mehr im Allgemeinen. Ein einfaches Gegenbeispiel liefern die Inschriften von X 4 / 4 Und X 6 / 6 . Einzelheiten siehe Kiselman, "Smoothness of Vector Sums of Plane Convex Sets", Math. Scannen. 60 (1987), 239–252.

20 / 3 ? Was ist das für eine Magie!? +1 interessantes Beispiel.

ich ich ist eingebildet.            

Ich erinnere mich, dass ich es erstaunlich fand, als ich diese Berechnung zum ersten Mal durchführte.
Glauben ich ich wohldefiniert ist, ist illusorisch.
Ich sage sicherlich nichts Tiefgründiges. Man sollte einfach keine Dinge aufschreiben, die wie konstante Ausdrücke aussehen, es aber nicht sind.
ich ich 0,207879
@bof: Nun, um für mich "offensichtlich wahr" zu sein, muss offensichtlich Sinn gemacht werden. Jedenfalls würde ich das nicht für "offensichtlich wahrer" halten als " ich × ich ist eingebildet".
Keiner der unendlich vielen Werte für ich ich sind frei erfunden :)
@bof: Ja. Und das zu sagen " ich ich ist imaginär" ist falsch, selbst wenn man die mehrwertige Natur berücksichtigt . Das meinte ich.
@xfix Ähm, nein ist es nicht.
@anorton Hängt davon ab, wie Sie "imaginär" definieren. Die übliche Definition ist "mit Realteil 0". und 0 hat sicherlich Realteil 0.
@Snowbody Ah. Ich glaube, ich habe diese Definition ziemlich oft gesehen. Ich denke normalerweise an "imaginär" wie in der Menge C R (Daher meine Verwirrung)
@Snowbody Die übliche Definition von "imaginär" ist "einen imaginären Teil ungleich Null". Die Definition von "rein imaginär" ist "mit Realteil Null".
@AnonymousPi ich = exp ( ( 4 N + 1 ) π ich 2 ) Wo N N . Somit ich ich = exp ( ( 4 N + 1 ) π 2 ) .
@bof Ich dachte, "einen Imaginärteil ungleich Null zu haben" sei die Definition von nicht real. Manche Zahlen sind real, manche Zahlen nicht. Einige Zahlen sind imaginär, andere nicht. Die reellen Zahlen, imaginären Zahlen und andere sind alle komplex. Wikipedia stimmt mir zu, bei allem, was es wert ist ... (nicht viel)
reine imaginäre Zahlen und reelle Zahlen bilden eine Richtungssummenzerlegung der komplexen Zahlen. Damit dies natürlich ist, muss Null sowohl real als auch rein imaginär sein.
Ist das wirklich intuitiv eingebildet? Könnte man nicht ahnen, dass es komplex mit einem Realteil ungleich Null und daher nicht imaginär ist? Oder gibt es vielleicht keine Intuition und man muss nur eine einfache Berechnung anstellen?
@Dr.MV Wie kann es negativ sein?
@AkivaWeinberger Haben wir
ich ich e ich Protokoll ( ich ) = e ich × ich ( π / 2 + 2 k π ) = e π / 2 2 k π
So, ich ich ( 0 , )
... Und ich ich ist rational

Dieser Teil ist wahr (Jordan-Brouwer-Trennsatz):

(a) Jede Einbettung der 2 -Sphäre hinein 3 -dimensionaler euklidischer Raum trennt den Raum in zwei disjunkte Bereiche.

Aber dieser Teil, der eine natürliche Verallgemeinerung des Jordan-Schönflies-Kurvensatzes zu sein scheint, ist nicht wahr:

(b) Die Regionen sind homöomorph zum Inneren und Äußeren der Einheitskugel.

Ich habe selbst an dieses Beispiel (die gehörnte Kugel) gedacht. Meine Antwort kann in gewisser Weise als eine Art 2D-Analog angesehen werden ...
Dies ist das erste Ergebnis in diesem Thread, das ich eigentlich nicht intuitiv finde. Was zum Teufel?

Ich mag "falsche Beweise" sehr, da Ihnen normalerweise die Einsicht, warum der Beweis falsch ist, ein gewisses Verständnis des Themas vermittelt. Eine sehr einfache Version ist diese, die ich in meinen ersten Semestern als Tutor geworfen habe:

Jede binäre Relation, die symmetrisch und transitiv ist, ist auch reflexiv und damit eine Äquivalenzrelation.

"Nachweisen":

Lassen eine symmetrische und transitive Relation bezeichnen und sei X , j seien zwei Elemente mit X j . Als symmetrisch ist, gilt das j X . Seit X j Und j X folgt aus der Transitivität von Das X X , was die Definition von Reflexivität ist.

Bearbeiten: Da ich gefragt wurde, ist hier der Grund, warum der Beweis falsch ist (bewegen Sie die Maus dorthin, um es anzuzeigen):

Betrachten Sie die leere Relation auf einer nicht leeren Menge S , so dass es keine gibt X , j S so dass X j . Diese Relation ist symmetrisch und transitiv, aber nicht reflexiv. Reflexivität braucht X X für alle halten X . Der Beweis geht davon aus, dass es ay gibt, so dass x ~ y, was nicht unbedingt für alle gilt X .

Wo ist der Fehler in dieser Argumentation?
@ user21820 Entschuldigung, für die zu zweideutige Formulierung. Ich habe es bearbeitet und es sollte jetzt klar sein, was gemeint ist.
Entschuldigung, ich habe mich verlesen.
Eine Beziehung ist nicht einfach „reflexiv“; vielmehr ist es auf einem Satz reflexiv. Die Beziehung An { 1 , 2 , 3 , 4 } woher 3 3 , 4 4 , 3 4 , Und 4 3 , und nichts anderes ist verwandt, ist am Set nicht reflexiv { 1 , 2 , 3 , 4 } , aber es gibt tatsächlich eine Menge, auf der es reflexiv ist.
Diese Situation wird ausführlich auf Seite 30 des Buches ESSENTIALS OF ABSTRACT ALGEBRA von Bundrick und Leeson (1972) behandelt.
Die symmetrischen transitiven Beziehungen umfassen eine sehr wichtige Klasse von Beziehungen, die manchmal als partielle Äquivalenzbeziehungen bezeichnet werden . Genauso wie Sie durch eine Äquivalenzbeziehung modifizieren können, um einen Quotienten zu erhalten, können Sie sich auf eine partielle Äquivalenzbeziehung beschränken und durch eine partielle Äquivalenzbeziehung modifizieren, um einen Subquotienten zu erhalten, dh eine Teilmenge eines Quotienten (oder äquivalent einen Quotienten einer Teilmenge). …

Hier ist einer meiner Favoriten: Nehmen wir an, wir spielen mit einer fairen Münze.

Satz (falsch) Bei einem langen Münzwurfspiel wird jeder Spieler etwa die Hälfte der Zeit auf der Gewinnerseite stehen, und die Führung wird nicht selten von einem Spieler auf den anderen übergehen.

Das Folgende ist aus W. Fellers Klassiker Einführung in die Wahrscheinlichkeitstheorie und ihre Anwendungen , Band 1:

Nach weitverbreiteter Meinung soll ein sogenanntes Gesetz der Mittelwerte den obigen Satz sicherstellen . Tatsächlich ist dieser Satz jedoch falsch und entgegen der üblichen Annahme gilt:

Mit Wahrscheinlichkeit 1 2 Unabhängig von der Spieldauer kam es in der zweiten Spielhälfte zu keinem Ausgleich. Außerdem sind die Wahrscheinlichkeiten in der Nähe des Endpunkts am größten .

Tatsächlich führt dies zum Arcus-Sinus-Gesetz für letzte Besuche (siehe zB Band 1, Kap. 3, Abschnitt 4, Theorem 1).

Hinweis: Bitte beachten Sie die bemerkenswerten Aussagen aus Kapitel III: Fluktuationen beim Münzwurf und Irrfahrten :

Beispielsweise wird in verschiedenen Anwendungen angenommen, dass Beobachtungen an einem einzelnen Münzwurfspiel während eines langen Zeitintervalls die gleichen statistischen Eigenschaften ergeben wie die Beobachtung der Ergebnisse einer großen Anzahl unabhängiger Spiele zu einem gegebenen Zeitpunkt. Das ist nicht so.

und später:

Dennoch liegt es auf der Hand, dass, wenn selbst das einfache Münzwurfspiel zu paradoxen Ergebnissen führt, die unserer Intuition widersprechen, diese in komplizierteren Situationen nicht als verlässlicher Leitfaden dienen kann.

[16.07.2015] Laut einem Kommentar von @HenningMakholm zeigen einige Beispiele auffällige Aspekte.

  • Nehmen wir an, dass ein ganzes Jahr lang Tag und Nacht sehr viele Münzwurfspiele gleichzeitig mit einer Rate von einem pro Sekunde durchgeführt werden. Im Durchschnitt findet in einem von zehn Spielen der letzte Ausgleich vorher statt 9 Tage vergangen sind, und die Führung ändert sich während der folgenden 356 Tage nicht. In einem von zwanzig Fällen findet der letzte Ausgleich innerhalb statt 2 1 2 Tagen, und in einem von hundert Fällen tritt es innerhalb des ersten auf 2 Stunden und 10 Protokoll.

  • Nehmen wir an, dass ein Kind in einem Lernexperiment, das ein Jahr dauert, ständig hinterherhinkt, außer vielleicht in der ersten Woche. Ein anderes Kind war beständig voraus, außer vielleicht in der letzten Woche. Würden die beiden Kinder als gleich beurteilt werden? Doch lassen Sie eine Gruppe von 11 Kinder einem ähnlichen Lernexperiment ausgesetzt werden, das keine Intelligenz, sondern nur Zufall beinhaltet. Einer unter den 11 würde für alle bis auf eine Woche als Anführer auftreten, ein anderer für alle bis auf eine Woche als Nachzügler.

Die obigen Beispiele sind tatsächlich eine Folge des Arcus-Sinus-Gesetzes für letzte Besuche .

Könnten Sie dem dritten Zitat eine Erklärung hinzufügen, wie es mit der üblichen Kurzschriftannahme koexistieren kann, dass ein "Münzwurf" unabhängig von jedem anderen Münzwurf irgendwo in der Raumzeit ist? Wie würden die im langen Spiel beobachteten Münzen "wissen", dass sie in einem langen Spiel verwendet werden, anstatt gleichzeitig in verschiedenen Spielen ausgeführt zu werden?
@HenningMakholm: Ich werde einige Informationen hinzufügen. Tatsächlich reduziert es sich hinter den Kulissen auf einen rein kombinatorischen Aspekt. Es gibt mehr Möglichkeiten, die Höhe im Inneren zu erhalten oder zu erhöhen k Schritte, als nach unten zu gehen. Vielleicht könnte man informell sagen, dass es nicht die Münze ist, die etwas über ihren Weg weiß, sondern der umgebende Raum mit der spezifischen Gitterstruktur, der ein gewisses Wissen über die eingebettete Münze und ihre aktuelle Höhe hat. :-)
Die wahrscheinlich am meisten benötigten Informationen sind die "Beobachtungen", über die sie sprechen. Wie zitiert, klingt es so, als würden sie behaupten, Sie könnten ihnen eine Liste von 1000 Münzwürfen geben, die entweder aus einem langen Spiel oder aus 1000 verschiedenen Spielen stammen, und sie könnten irgendwie mit einer Wahrscheinlichkeit von mehr als 1/2 erraten, welche der statistischen Analyse.
@HenningMakholm: Ich habe zwei Beispiele hinzugefügt, die hilfreich sein könnten. Mit freundlichen Grüßen,
Beachten Sie, dass "Beobachtungen eines einzelnen Münzwurfspiels während eines langen Zeitintervalls die gleichen statistischen Eigenschaften ergeben wie die Beobachtung der Ergebnisse einer großen Anzahl unabhängiger Spiele zu einem bestimmten Zeitpunkt." kann tatsächlich (etwas) wahr sein, wenn Sie es richtig machen (dh indem Sie exponentielle Zeitskalen betrachten). Es kommt von der Pfadkonvergenz zu einer Brownschen Bewegung, Skaleninvarianz der Brownschen Bewegung und Ergodizität/Mischen des Neuskalierungsflusses der Brownschen Bewegung.
D.Thomine: Interessante Informationen. Danke!
Können Sie bitte einen Verweis auf das Buch " Chung & Feller Introduction to Probability Theory and It's Applications, Vol 1 " geben, das Sie vorschlagen? Ich kenne das gleichnamige klassische Buch , aber es ist nur von Feller, und ein anderes ist Elementary Probability Theory von Chung und AitSahlia.
@ApoorvPotnis: Es tut mir leid, das war ein Fehler. Ich beziehe mich auch auf das Buch von W. Feller. Das Arcus-Sinus-Gesetz wurde zuerst von KL Chung und W. Feller bewiesen und anscheinend habe ich es verwechselt. Ich habe den Beitrag aktualisiert. Mit freundlichen Grüßen,

Theoreme, die intuitiv wahr, aber tatsächlich fehlerhaft sind:

  • Es gibt keine stetige, nirgendwo differenzierbare reelle Funktion.

  • Es gibt keine reelle Funktion, die auf einem nicht-trivialen Intervall differenzierbar und nicht monoton ist.

  • Wenn eine reelle Funktion erfüllt X , j , F ( X + j ) = F ( X ) + F ( j ) , es ist von der Form X A X .

  • Unendliche Summen und Integrale können jederzeit ausgetauscht werden.

  • Ein zusammenhängender metrischer Raum ist wegzusammenhängend.

Warum ist intuitiv wahr, dass unendliche Summen und Integrale jederzeit vertauscht werden können? Ich denke, es ist relativ einfach, ein Gegenbeispiel zu finden.
Können Sie ein Gegenbeispiel für die dritte Behauptung zeigen? (Der über lineare Funktionen)
Fast jede Aussage, bei der das Gegenbeispiel mit „Let H sei eine Hamel-Basis von R …“ wird eines dieser Schein-wahr-ist-nicht-Dinge sein.
Ich habe das Gefühl, dass die über eine lineare Funktion nicht wirklich passt. Ist die "lineare Funktion" nicht durch diese Eigenschaft plus definiert ? F ( A X ) = A F ( X ) ? Wäre es nicht nur für jemanden nicht offensichtlich, der die Verwendung der "linearen Funktion" falsch interpretiert?
@ jpmc26 ist nicht die Definition F ( A X + B j ) = A F ( X ) + B F ( j ) oder F ( A X ) = A F ( X ) Und F ( X + j ) = F ( X ) + F ( j ) ?
"Es gibt keine wirkliche Funktion, die in einem nicht-trivialen Intervall differenzierbar und nicht monoton ist." Ernsthaft? Was ist das Gegenbeispiel?
@Alessandro Ich sagte "diese Eigenschaft plus ...", um mich auf die in der Antwort zu beziehen.
Für Nr. 4 ist die Einschränkung, wo dies funktioniert, durch das Fubini-Tonelli-Theorem gegeben

Folgende Aussage hielt ich mal für "offensichtlich":

Wenn F : R [ 0 , ) kontinuierlich ist so, dass F ( X ) D X < , Dann lim X ± F ( X ) = 0

was eigentlich falsch ist.

(Anmerkung: Es ist wahr, wenn F ist gleichmäßig stetig!)

Können Sie ein Gegenbeispiel nennen?
Dies ist ziemlich einfach, um ein Gegenbeispiel zu geben: let F Unebenheiten so weit unten haben, wie Sie möchten, nur mit ausreichend kleinen Breiten, damit die Summe ihrer Integrale gegen Null geht. Du kannst das schaffen lim sup X F ( X ) = Hier entlang.
@ hallaplay835 vgl. Stein & Shakarchis Real Analysis-Buch, das Kapitel über L 1 Funktionen. Es war eine Übung.
@hallaplay835 Wenn F ( X ) = cos ( X 2 ) , dann klar F hat nicht einmal eine Grenze als X nähert sich der Unendlichkeit. Aber das Integral von F existiert als uneigentliches Riemann-Integral (Hinweis: Variable ändern j = X 2 Und D X = 1 2 ( j ) und Abels Test erledigt den Rest!).
Sie können davon ausgehen F positiv sein. Das macht die Bedingung etwas irreführender. Tatsächlich wird genau dies in vielen Büchern über Quantenmechanik behauptet, um den Zerfall der Verteilungsfunktion im Unendlichen zu beweisen.

Es gibt eine ganze Reihe kontraintuitiver Wahrscheinlichkeitssituationen da draußen. Einer meiner Favoriten sind nichttransitive Würfel:

Es gibt 3 Würfel, A, B und C. Die Würfel haben Zahlen von 1-9 auf ihren Seiten (Wiederholungen möglich). Wenn Würfel B (höhere Zahl) Würfel A mehr als die Hälfte der Zeit schlägt und Würfel C Würfel B mehr als die Hälfte der Zeit schlägt, dann schlägt Würfel C Würfel A mehr als die Hälfte der Zeit.

Dies ist nicht unbedingt eine wahre Aussage. Würfel können so entworfen werden, dass die Eigenschaft "x schlägt y" nicht transitiv ist. A schlägt B, das C schlägt, das A schlägt.

Es gibt eine Reihe ähnlich kontraintuitiver Aussagen bei der Mehrwegabstimmung, von denen dies als Sonderfall angesehen werden könnte. Beispielsweise gibt es bei Eiskunstlaufwettbewerben auf Meisterschaftsebene neun Richter. Es kam manchmal vor, dass eine Mehrheit der Richter Skater einstuften A über Skater B , würde eine Mehrheit rangieren B über C , und eine Mehrheit würde Rang C über A . Die Bewertungsregeln mussten Bedingungen enthalten, um mit dieser Art von Situation fertig zu werden.
Tatsächlich können Sie unter den gewünschten Kandidaten einen beliebigen verrückten gerichteten Graphen finden, und es gibt eine Population von scheinbar rationalen Wählern linearer Ordnung, deren Wahlverhalten zu diesem Ergebnis führt. Es stellt sich heraus, dass das EINZIGE Verhalten für Wähler, das zu konsistenten Mehrheitsentscheidungen führt, darin besteht, blind einer Parteilinie zu folgen.
Ist neun die Mindestanzahl von Seiten/Richtern, die erforderlich ist, damit das Nicht-Transitivitätsphänomen erreicht werden kann?
Definitiv nicht. Die meisten nicht transitiven Würfel werden als 6-seitig beschrieben. Es ist nur so, dass die Zahlen auf ihnen von 1-9 sind (nicht unbedingt alle verwendet).
@Duncan Was ist die Mindestseitenzahl für die Würfel? würden Sie bitte ein Beispiel für diese Abstimmung nennen?
Die Mindestanzahl der Seiten wäre 3 (obwohl es schwierig ist, einen 3-seitigen Würfel zu haben). Aber nimm trotzdem diese als Beispiele: Würfel A: 2,4,9 Würfel B: 1,6,8 Würfel C: 3,5,7 Würfel A schlägt Würfel B 5/9 der Zeit, Würfel B schlägt Würfel C 5/ In 9 der Fälle schlägt Würfel C in 5/9 der Fälle Würfel A. (Dies ist nur das Beispiel von der Wikipedia-Seite zu nicht-transitiven Würfeln, aber mit halbierter Seitenzahl.)
@duncan Angenommen, Sie haben drei Skater, XY und Z. Richter 1 bevorzugt X gegenüber Y und Y gegenüber Z; Richter 2 bevorzugt Y gegenüber Z und Z gegenüber X; Richter 3 bevorzugt Z gegenüber X und X gegenüber Y. Jetzt bevorzugen zwei von drei Richtern X gegenüber Y; zwei bevorzugen Y gegenüber Z und zwei bevorzugen Z gegenüber X.

Die reellen Zahlen/Cantor-Menge sind abzählbar.

Es gibt mehrere falsche "offensichtliche" Beweise:

  1. „Beweis“ . Betrachten Sie den Baum { 0 , , 9 } N , dann entspricht jede reelle Zahl einem Knoten im Baum. Da es nur abzählbar viele Ebenen gibt und jede endlich ist, folgt daraus, dass die reellen Zahlen endlich sind.

    Warum scheitert es? Dieses Set ist eigentlich kein Baum. Sie können es so anordnen, dass es wie ein Baum aussieht, aber tatsächlich würde der Baum aus Anfangssegmenten jeder Funktion bestehen, die nach Fortsetzung geordnet sind. Dieser Baum hätte dann eine letzte Ebene (nämlich eine Ebene, auf der kein Punkt dort einen Nachfolger hat), und es wäre genau die Ebene der Funktionen selbst (die vorherigen Ebenen wären echte Anfangssegmente der Funktionen).

    Wenn wir diese letzte Ebene entfernen, ist der Baum tatsächlich zählbar, aber jetzt entspricht jede reelle Zahl einem Zweig im Baum und nicht mehr einem Knoten. (Es ist der eindeutige Zweig, dessen Limit gleich der Funktion ist, die zuvor auf dieser letzten Ebene aufgetreten ist.)

  2. „Beweis“ . Die rationalen Zahlen sind abzählbar, und zwischen jeweils zwei reellen Zahlen gibt es eine rationale Zahl. Daher definiert dies eine Bijektion zwischen Paaren von reellen Zahlen und den rationalen Zahlen.

    Warum scheitert es? Da es viele, viele, viele Paare gibt, die auf dieselbe rationale Zahl abgebildet werden, ist dies eigentlich keine Bijektion.

  3. „Beweis“ . Die Cantor-Menge ist abgeschlossen, ihr Komplement ist offen, also ist sie eine abzählbare Vereinigung von Intervallen, also ist die Cantor-Menge abzählbar.

    Warum scheitert es? Denn nicht jeder Punkt in der Cantor-Menge ist Endpunkt eines solchen Intervalls. Zum Beispiel 1 4 . Allerdings bilden die Endpunkte dieser Intervalle eine abzählbare dichte Teilmenge.

  4. BONUS! , P ( N ) ist zählbar.

    „Beweis“ . Für jede Endlichkeit N , P ( N ) ist endlich, und P ( N ) = P ( N ) = P ( N ) , ist eine abzählbare Vereinigung endlicher Mengen, die abzählbar ist.

    Warum scheitert es? Da die Vereinigung nur endliche Teilmengen von enthält N , aber keine seiner unendlichen Teilmengen.

Ich denke, dass die Nichtexistenz irrationaler Zahlen selbst in diese Kategorie fallen könnte (offensichtlich wahr, aber falsch), aber vor so langer Zeit als falsch entdeckt wurde, dass wir uns nicht mehr daran erinnern, wie bizarr es ausgesehen haben muss.
@MJD: Stimmt, und in zwei Jahrhunderten werden sie es seltsam finden, dass die Leute das irgendwann gedacht haben R zählbar sein kann. Oder zumindest das, was ich hoffe!
Jetzt kommt es mir seltsam vor, dass niemand vor Cantor das beobachtet hat, wenn R zählbar wären, könnte es von einer Familie disjunkter Intervalle überdeckt werden, deren Gesamtlänge kleiner als irgendeine gegebene Länge wäre ϵ .
@MJD: Hmm, es ist ein bisschen seltsam. Vielleicht haben sie all die Leute hingerichtet, die das vorgeschlagen haben, bevor sie ihre Ergebnisse veröffentlichen konnten! ;-)
In welchem ​​Sinne hat Ihr Baum eine „Endebene“ und was bedeutet dieser Begriff hier überhaupt?
@Christopher: Genau genommen ist es nicht einmal ein Baum. Aber bei der Präsentation dieses Beweises wird oft argumentiert, dass der Baum die Anfangssegmente einer Funktion bildet N Zu { 0 , , 9 } . Die letzte Ebene ist die Funktion selbst. Und es bedeutet, dass diese Knoten keine Nachfolger haben (und sie selbst keine Nachfolger irgendeines anderen Knotens sind).
Da die Dezimalerweiterung von beispielsweise 1 / 3 , hat keine Endziffer, der „Baum“ hat in diesem Sinne keine letzte Ebene.
@Christopher: Nein, der Baum hat ω + 1 Ebenen. Das letzte ist die unendliche Folge 0,333 .
Damit habe ich meinen Seelenfrieden geschlossen P ( N ) ist vor langer Zeit unzählbar. Tatsächlich ist es so tief in meinem Gedächtnis verankert, dass es mir schwer fällt zu akzeptieren, dass die Menge endlicher Teilmengen von N ist zählbar!
@triple_sec: Haha, ja, das kann auch passieren. Obwohl es eines dieser Dinge ist, die, wenn man eine Weile darüber nachdenkt, wieder deutlich werden.
@Asaf Urkomisch, mit deinem ω + 1 Sie haben die Idee des weit verbreiteten Missverständnisses verwendet " 0,999 < 1 weil sie sich um einen Betrag unterscheiden 0,000 1 “ in einem korrekten Argument über einen anderen, eng verwandten, bekanntermaßen bezweifelten Satz.
@Ryan: Ich verstehe nicht wie. Die Objekte im Baum sind keine reellen Zahlen. Sie sind Funktionen von N Zu { 0 , , 9 } . Wir können sie als reelle Zahlen identifizieren, aber diese Identifizierung muss nicht eins zu eins sein; einfach drauf. Und genau darum geht es hier. Auch die Elemente auf der ω + 1 ist keine Folge vom Auftragstyp ω + 1 , sondern eine Reihenfolge des Auftragstyps ω .
@Asaf: Ich meinte nicht, dass Sie tatsächlich Dezimalfolgen des Typs verwendet haben ω + 1 , sondern einfach, dass Sie reelle Zahlen als Dezimalzahlen über einen induktiven Prozess mit konstruiert haben ω + 1 Schritte, wo jeder der erste ω ist eine endliche Dezimalzahl, auf die der Irrglaube richtig zutrifft.
@Ryan: Auch das habe ich nicht getan. Ich habe keine reelle Zahl konstruiert. Ich habe lediglich gesagt, dass es eine Möglichkeit gibt, jede reelle Zahl als Knoten im Baum zu identifizieren. Einige haben möglicherweise mehr als einen Weg.
@Asaf, ich sehe nicht, wie Sie den Auftragstyp erhalten ω + 1 Einträge in Elementen der Menge { 0 , , 9 } N . Ich denke zum Beispiel, wir stimmen darin überein, dass es genau ein Element in dieser Menge gibt, das der Dezimalerweiterung von entspricht π 3 . Wenn dieses Element, wie Sie sagen, einen Auftragstyp hat ω + 1 , dann hat es eine letzte Ziffer. Welche ist das und warum? Und wie kommt es zu einer Funktion aus N Zu { 0 , , 9 } ?
@Christopher: Die Höhe des Baumes ist ω + 1 . Nicht ω . Um explizit zu sein, lassen Sie mich genau definieren, was der Baum ist, der N -te Ebene (wo N < Ö M e G A + 1 ) ist genau die Menge von F N . Und S < T Wenn S T (was bedeutet, dass beides als Einschränkung derselben Funktion angesehen werden kann). Das Element hat keinen "Auftragstyp" und ich denke, dass Sie hier etwas ganz Grundlegendes missverstehen, aber ich bin mir nicht sicher, was genau.
@Asaf, vielleicht könntest du hier klarstellen, was genau du mit "Höhe" meinst. Jedes Element der Menge, die Sie definiert haben, hat eine eindeutige Reihenfolge ω , und obwohl ich glaube, dass wir dasselbe für „the N -das Niveau“ wann N ist endlich, wenn ich lese F N als F auf den Preimage-Satz beschränkt { 0 , , N } . Nun, nichts davon ist tatsächlich in Ihrem Set. Jedes Element Ihrer Menge ist die Vereinigung einer zählbaren Anzahl solcher endlicher Folgen, die Sie als endliche Pfade betrachten können. Aber was wäre mit Ihrer Definition von „Höhe“ ein solcher Baum der Höhe? ω ?
@Christopher: Ja, genau das steht in meiner Antwort, wenn Sie es lesen möchten. Der Satz { 0 , , 9 } N ist KEIN Baum, aber wir können es uns als eine Menge vorstellen, die sich in gewissem Sinne ein bisschen wie ein Baum verhält. Aber der eigentliche Baum ist die Beschränkung jeder Funktion auf ein Anfangssegment der Domäne. Es gibt ω richtige Anfangssegmente, und dann gibt es noch ein Anfangssegment, das die eigentliche Funktion ist. Dies definiert einen Baum, der Höhe hat ω + 1 . Und mit der Höhe des Baumes meine ich die Ordnungsart seiner Ebenen, da es welche gibt ω + 1 Ebenen, das ist die Höhe.
@MJD War das nicht die Beobachtung R ist Abzählbarkeit für Cantors Beobachtung nicht notwendig?
@Brilliand: Das war Cantors Beobachtung.
All diese Verwirrung um den Nicht-Beweis 1. Können wir nicht einfach fragen, welcher natürlichen Zahl das entsprechen soll? π ?
@Cory: Oder 1 3 , Ja. Es gibt viele Beispiele. Aber das erklärt nicht ganz, warum der Beweis fehlschlägt. Es zeigt nur, dass es geht. Ich habe versucht, den Grund für das Scheitern tatsächlich zu erklären.

Hypothese: Jede unendlich differenzierbare Funktion ist irgendwo reell-analytisch.

Dies ist falsch, wie (zum Beispiel) die Fabius-Funktion zeigt .

Sehen Sie sich diese interessante Antwort von Dave L Renfro und die Beiträge an, auf die sie verweist.

Ich bin überrascht, dass noch niemand diese Antwort gegeben hat, also hier ist sie:

Es gibt mehr ganze Zahlen als natürliche Zahlen.

Es ist offensichtlich, nicht wahr?

Ich zögere mit diesem, weil es von der (etwas unsoliden) Definition von "mehr" abhängt.
Es gibt absolut mehr ganze Zahlen als natürliche Zahlen (durch Einbeziehung). Und Kardinalität ist ein leicht erfundener Begriff, wenn Sie einen Moment darüber nachdenken.
@Tibor: Nein, Kardinalität ist kein erfundener Begriff, wenn man lange darüber nachdenkt .
Die gesamte Mathematik ist per Definition erfunden. :)
„Ich bin überrascht, dass noch niemand diese Antwort gegeben hat“. Tatsächlich wurde der vierte Unterabsatz dieser Antwort „Es gibt genauso viele gerade Zahlen wie natürliche Zahlen.“ am 5. Juni um 13:58 Uhr gepostet (implizit listet er Dinge auf, die wahr, aber nicht offensichtlich sind), und diesen Kommentar , „… es gibt halb so viele ungerade ganze Zahlen“ [als ganze Zahlen], wurde am 6. Juni um 7:30 Uhr gepostet.
Diese sind im Wesentlichen äquivalent zu Ihrem Post: die Behauptung, dass, wenn eine Eins-zu-Eins-Korrespondenz zwischen einer (nicht leeren) Menge und einer strengen Teilmenge (insbesondere „der Hälfte“) einer anderen (nicht leeren) Menge besteht, dann es kann keine Eins-zu-Eins-Entsprechung zwischen den Sätzen in ihrer Gesamtheit geben. Wie ich in einem anderen Kommentar erwähnt habe, glaube ich, dass dies wahr ist, wenn die Mengen endlich sind, was es offensichtlich macht.
Ich stimme GOTO o zu, weil nicht nur die Tatsache, dass ganze Zahlen grundsätzlich ein besserer Name für ganze Zahlen sind und natürliche Zahlen nur positive ganze Zahlen sind, sondern auch weil Sie berechnen können, wie viele ganze Zahlen sie im Vergleich zu natürlichen Zahlen sind, wenn wir i sein lassen die Anzahl der ganzen Zahlen und n die Anzahl der natürlichen Zahlen und wir erhalten ... i = 2n + 1 und n = (i - 1)/2 ... weil es so viele natürliche Zahlen wie negative ganze Zahlen gibt (was im Grunde die ist gegenüber und es muss die gleiche Anzahl von Gegensätzen wie bei regulären Zahlen geben) + 0, was zwei n plus 0 ist, was als zusätzliche Zahl zählt.
Zunächst war es seltsam, dass es genauso viele natürliche Zahlen wie ganze Zahlen gibt. Aber im Laufe der Jahrhunderte hat man sich daran gewöhnt, bis es schockierend war, dass es tatsächlich mehr reelle als rationale Zahlen gibt!

Das Bild eines Maß-Nullsatzes unter einer kontinuierlichen Karte hat das Maß Null!

Ist das nicht wahr?!?!
@LeilaHatami Nein, siehe hier . Im Wesentlichen können wir die Cantor-Menge bijektiv (und kontinuierlich) auf [0,1] abbilden, was eine Takt-0-Menge auf eine Takt-1-Menge abbildet.

Die Wahrscheinlichkeit, dass Sie einen beliebigen Punkt auf einer Dartscheibe treffen, ist 0 aber die Wahrscheinlichkeit, dass Sie die Dartscheibe treffen, ist 1 (solange du nicht so schlecht Darts werfen kannst wie ich ;D).

BEARBEITEN:

Wie @JpM betonte, habe ich das Format dieser Beiträge nicht befolgt, obwohl die Idee ( meiner Meinung nach leicht ) aus dem, was ich oben gesagt habe, verstanden werden kann.

Pseudo-Claim: Die Wahrscheinlichkeit, einen einzelnen Punkt auf einer Dartscheibe zu treffen, ist größer als 0 da die Wahrscheinlichkeit, es überhaupt zu treffen (vorausgesetzt, Sie treffen die Dartscheibe), hoch ist 1 .

Scheint offensichtlich in dem Sinne, dass ein Haufen von 0 kann sich nicht summieren 1 also muss jeder Punkt eine gewisse Wahrscheinlichkeit haben. Eigentlich falsch wegen einiger Eigenschaften von Maßen.

... ist dies nicht ein nicht offensichtliches Theorem, das tatsächlich wahr ist??
@JpMcCarthy Laien könnten behaupten: "Die Wahrscheinlichkeit, dass wir einen einzelnen Punkt treffen, ist größer als 0 „Dann ist dies tatsächlich ein Satz, der eigentlich falsch ist.
Insbesondere gibt es eine "offensichtliche" Intuition, dass der Punkt, den Sie tatsächlich getroffen haben, eine Wahrscheinlichkeit größer als gehabt haben muss 0 , "seit Wahrscheinlichkeit 0 Ereignisse sind unmöglich". Ich denke, dass dieses Beispiel so offensichtlich ist, dass es in der Theorie axiomatisch falsch gemacht werden muss, um es aus dem Weg zu räumen und etwas Arbeit zu leisten :-)
Was ist, wenn ich behaupte, dass die Wahrscheinlichkeit, sie zu treffen, eine nicht-archimedische hyperreelle Zahl ist?
@DanielV Ich würde aufgrund meiner Stimmung eines von zwei Dingen sagen: a) Ein Laie würde so etwas nicht behaupten, dh er würde nicht sagen, dass die Wahrscheinlichkeit, einen einzelnen Punkt zu treffen, unendlich klein ist. b) Generell (habe ich eigentlich noch nicht anders gesehen, dann bin ich wieder jung und naiv) werden Maße mit Werten in den erweiterten reellen Zahlen definiert, also ohne Infinitesimale.
Ich bin verwirrt. Vielleicht wollen Sie damit sagen: "Die Wahrscheinlichkeit, dass Sie einen Punkt auf einer Dartscheibe treffen, ist nicht Null"?? Das wäre offensichtlich und falsch.
@JpMcCarthy genau das habe ich im Kommentar gesagt.
...dann sollte das deine Antwort sein! Ihre Antwort ist eher ein nicht offensichtliches Theorem, das tatsächlich wahr ist, als ein offensichtliches Theorem, das tatsächlich falsch ist.
@JpMcCarthy ok, ich habe das Format meines Beitrags geändert, um genau zu verfolgen, wie die Frage gestellt wurde.
@JpMcCarthy Danke; Ich mag die über meinen mehr, aber das war interessant.
Sollte angenommen werden, dass "Punkt" "Bereich auf der Dartscheibe so bedeutet, dass die Fläche 0 ist" (oder vielleicht "0-dimensionaler Bereich auf der Dartscheibe", obwohl die Eigenschaft auch für eine 1-dimensionale Linie gelten würde ), was nicht unbedingt mit der Laiendefinition von "Punkt" übereinstimmen würde?
@JAB Ich denke, Sie verkomplizieren das zu sehr, ein Laie wird dies nicht eingehend über die Definition von "Punkt" nachdenken. Ich betrachte einen Punkt als genau das, was ein Laie denken würde, eine genaue Stelle auf der Dartscheibe.
Ich denke, das Wesentliche, was manche Leute einfach nicht verstehen können, ist, dass ein Ereignis mit einer Wahrscheinlichkeit von 0 immer noch passieren kann.
@Neil ja, genau das wollte ich hier erreichen ;D
Dies ist weder eine mathematische Aussage, noch ist sie falsch.
Ist das nicht analog dazu, dass ein Integral von a nach a eines integrierbaren f für 0 < a < 1 > 0 sein muss, wenn das Integral von f von 0 nach 1 nicht Null ist?
@scibuff ist bei richtiger Einstellung tatsächlich äquivalent

Mein "Theorem":

Die Aussage Jeder liebt mein Baby, aber mein Baby liebt niemanden außer mir handelt von einem Liebespaar

Es ist so einfach und so offensichtlich, sogar meine Oma wird es verstehen. Und egal, wie sehr Sie die einfache Logikrechnung erklären, die zeigt, dass wir hier von einem einzelnen Narzissten sprechen, die halbe Klasse der Logikstudenten im ersten Semester wird weiterhin darauf bestehen, dass Ihr Beweis falsch ist, und sie wissen nicht, was falsch ist darüber, aber es kann sich nicht auf eine einzelne Person beziehen.

Manche Menschen behandeln Liebe als eine Beziehung, die nicht reflexartig definiert wird.
@Joshua Ich muss noch einen Studenten treffen, der bemerkt, dass dies die Wurzel des Problems ist, und erklären, wie man es im logischen "Beweis" vermeidet. Sie haben natürlich recht, sobald wir die Menge „jeder“ nicht als „alle Menschen“, sondern „alle Menschen, die in der Lage sind, mein Baby zu lieben“ definieren und entscheiden, dass mein Baby nicht in der Menge ist, ist das Ergebnis anders .
Ich habe keine Ahnung, was diese Antwort zu sagen versucht.
@goblin Wie aus diesem logischen Beweis hervorgeht, ist die einzig mögliche Bedeutung der Aussage, dass mein Baby ich bin .

Betrachten Sie eine Funktion F : ( 0 , ) R das ist C in diesem Intervall. Auf den ersten Blick könnte man meinen, wenn lim ( F ) = 0 als X , Dann lim ( F ' ) = 0 als X . Dies ist jedoch falsch. Hier nur ein Gegenbeispiel:

F ( X ) = 1 X Sünde ( X 2 )

Wenn wir ferner die Bedingung hinzufügen, dass F auch monoton sein, es lassen sich immer noch Gegenbeispiele finden (allerdings recht pathologisch).

Ihr "Anderer" wurde bereits erwähnt.........
Ups, habe es dort nicht gesehen. Danke.
Könntest du eine geben C monoton, Gegenbeispiele? Vielen Dank!
Ja, Sie können eine bekommen, indem Sie hier ein wenig an den Antworten herumbasteln: math.stackexchange.com/questions/788813/…

Ein einfacher Bogen (homöomorphes Bild des geschlossenen Einheitsintervalls) in der Ebene hat 2 -dimensionales Lebesgue-Maß Null.

das ist offensichtlich richtig – nicht wahr?
@athos Es sollte offensichtlich sein, sonst wäre es keine gültige Antwort auf diese Frage. Fragst du, warum es eigentlich falsch ist? Es gibt wahrscheinlich eine einfachere direkte Konstruktion eines "fetten Bogens" in der Ebene, aber sie folgt aus einem allgemeineren Satz von RL Moore und JR Kline ( Auf der allgemeinsten Ebene geschlossene Punktmenge, durch die es möglich ist, eine einfache kontinuierlicher Bogen , Ann. of Math. (2) 20 (1919), 218-223), dass jeder (Homöomorph des Standards) Cantor-Satz in der Ebene in einem Bogen enthalten ist; Wenden Sie dies auf ein fettes Cantor-Set im Flugzeug an.
Danke für den Link ... ein bisschen schwierig für mich, werde es für die spätere Verdauung bookmarken :)

Jede reelle Zahl kann irgendwie berechnet werden.

Formeller:

Für jede reelle Zahl gibt es ein Programm endlicher Länge, das diese Zahl berechnet.

Da reelle Zahlen nicht zählbar sind, während berechenbare Zahlen zählbar sind, kann das einfach nicht der Fall sein.

Diese Einschränkung ergibt sich aus der Tatsache, dass wir festgefahren sind, Programme endlicher Länge zu verwenden. Programme mit unendlicher Länge können definiert werden, um (trivialerweise) jede reelle Zahl zu berechnen. Es gibt also einen Sinn, in dem alle reellen Zahlen berechnet werden können .

Nur nicht von Menschen. Beachten Sie, dass, da ein einzelnes Programm unendlicher Länge unendlich viel Speicher beanspruchen würde (und wir scheinen keine unendlichen Computer/Gehirne zu haben), die Mehrheit dieser Programme unendlicher Länge niemals bekannt, geschweige denn berechnet werden kann. Berechenbare Zahlen sind also nur solche Zahlen, die von einem Programm endlicher Länge berechenbar sind. Und die Menge der Programme endlicher Länge ist abzählbar.

Was ich persönlich denke, sollte es sein... Für jede nichttransdentale, reelle Zahl gibt es ein Programm endlicher Länge, das diese Zahl berechnet...
@BinaryFreak Ich glaube, die Nicht-Transzendenten sind alle algebraisch und daher berechenbar, also wäre das ein offensichtliches Theorem, das sich als wahr herausstellt. Interessanterweise ist zwar jede nicht berechenbare Zahl transzendent, aber einige Transzendente sind berechenbar ( π , e , und einige sich nicht wiederholende reelle Zahlen wie 0.123456789101112... ).

Satz: Sei F 1 ( X , j ) Und F 2 ( X , j ) seien zwei gemeinsame Wahrscheinlichkeitsdichten, von denen jede ihre hat X , j Komponenten positiv korreliert ( C Ö v 1 ( X , j ) > 0 , C Ö v 2 ( X , j ) > 0 ). Lassen F 3 = a F 1 + ( 1 a ) F 2 für einige die Mischungsdichte sein 0 a 1 . Dann C Ö v 3 ( X , j ) > 0 .

In Worten: Das Mischen von Populationen bewahrt das Korrelationszeichen. Mit anderen Worten: Wenn der durchschnittliche männliche MSE-Benutzer heller als der Mittelwert ist und wenn die durchschnittliche weibliche MSE-Benutzerin heller als der Mittelwert ist, dann ist der durchschnittliche MSE-Benutzer heller als der Mittelwert. Offensichtlich wahr.

FALSCH. Siehe Simpsons Paradoxon .

Vielleicht wird das folgende Beispiel die Offensichtlichkeit deutlicher machen. Baseball-Team A hat ein besseres Gewinn-Verlust-Verhältnis als das Team B im ersten Saisonteil. Dann gibt es einen Streik, und einige Spiele werden verpasst. Wenn die Saison wieder aufgenommen wird, Team A hat auch ein besseres Gewinn-Verlust-Verhältnis als Team B im zweiten Saisonabschnitt. Daher Mannschaft A hat ein besseres Gewinn-Verlust-Verhältnis als das Team B gesamt. (Falsch!) Baseball spielte 1981 eine solche Split-Saison, aber ich weiß nicht, ob das Paradoxon tatsächlich damals auftrat. Wahrscheinlich nicht, wie es erforderlich ist A Und B sehr unterschiedlich viele Spiele zu spielen.

Beanspruchen:

Wenn das Skalarprodukt zweier Vektoren 0 ist, dann sind sie linear unabhängig.

Mein Prof hat mir heute diese Frage gestellt und ich bin darauf hereingefallen.

haha, hat eine Sekunde gedauert.
kommt drauf an in welchem ​​bereich. sehr schön, aber eine Fangfrage. Ich fühle mich betrogen.
@ Lost1: Es hängt eigentlich nicht vom Feld ab: v 0 = 0 Und { v , 0 } ist nie unabhängig.
@JasonDeVito ah ... ich dachte, nimm Z 2 2 und der Vektor ( 1 , 1 ) ...
@Lost1 Gemäß der üblichen Definition muss das Skalarprodukt reellwertig sein (sicherlich nicht finite Feldwerte; sie sind keine geordneten Felder, und ohne Ordnung macht positive Bestimmtheit keinen Sinn. Und ohne das ist dieser Satz falsch aber hat auch keinen grund wahr zu sein!)
@RyanReich Nicht unbedingt. In der linearen Algebra (und der Zahlentheorie) ist es ziemlich üblich, innere Produkte zu berücksichtigen, die Werte im zugrunde liegenden Feld der betrachteten Vektorräume annehmen. Nur bei der Spezialisierung auf reell- oder komplexwertige Felder kommt positive Bestimmtheit ins Spiel. Seit jetzt haben wir vielleicht v v = 0 für v 0 , wird der Begriff isotroper Vektoren verwendet.

Meiner Meinung nach sind die interessantesten (aber manchmal auch nicht intuitiven) Ergebnisse in der Mathematik diejenigen, die einen Satz aussagen, der am Ende falsch ist, weil er tatsächlich in vielen Fällen gilt, außer in sehr wenigen oder sehr seltsamen Fällen. Mit anderen Worten, die "offensichtlichsten" falschen Theoreme sind für mich diejenigen, die sehr schwierige Gegenbeispiele haben.

Einige Beispiele:

  • Banach-Tarski: Es gibt eine strikte Teilmenge A des Euklidischen N -Ball B so dass man sich aufteilen kann A Und B in ebenso viele weitere Teilmengen, die durch Isometrien aufeinander abgebildet werden können. Dies zeigt, dass nicht alle Mengen messbar sind und dass es möglich ist, Partitionen durchzuführen, die das Maß nicht erhalten.

  • Nichtendlichkeit differenzierbarer Strukturen: Für R N mit N = 4 , gibt es unzählige verschiedene differenzierbare Strukturen.

  • Divergenz der Fourier-Reihe: Es existiert eine integrierbare Funktion auf [ π , π ] deren Fourierreihe überall divergiert. Dies ist äußerst ungewöhnlich, da für jede typische Funktion, die wir aufschreiben könnten, ihre Fourier-Reihe normalerweise an einem oder einer endlichen Anzahl von Punkten divergieren kann, aber wahrscheinlich überall sonst konvergieren wird.

Da fällt mir ein: eine Teilmenge von R N ist entweder geschlossen oder offen (aber nicht beides). Was natürlich nicht stimmt, denn die leere Menge ist offen und abgeschlossen. (Aber ich glaube auch nicht Q ist entweder geschlossen oder offen R ).
Sie brauchen kein seltsames Beispiel, um eine Teilmenge von zu finden R N das ist weder offen noch geschlossen; Nehmen Sie einfach einen geschlossenen Ball und entfernen Sie einen Grenzpunkt. oder nehmen Sie eine Folge von Punkten, die gegen eine Grenze konvergiert, aber lassen Sie die Grenze weg. Die Terminologie mag zu der Annahme führen, dass Mengen entweder offen oder abgeschlossen sind, aber nichts an den tatsächlichen Konzepten lässt dies so erscheinen.
Was ist ein Beispiel für eine Funktion mit überall divergierenden Fourier-Reihen?
@Ruslan, Das Beispiel ist nicht trivial und stammt von Kolmogorov. Eine frühere, fast überall abweichende Konstruktion war einer von Kolmogorovs ersten Ruhmesansprüchen. Dazu müssen Sie Google konsultieren, ich kenne das Beispiel nicht.
@Ruslan Google führt uns hierher zurück .

Ein "offensichtliches", aber falsches Theorem: Es gibt mehr offene Mengen R 2 (oder R N ) als es reelle Zahlen gibt.

Und in ähnlicher Weise haben wir diese Folgerung zur ersten Aussage: Es gibt mehr stetige Funktionen R R als es reelle Zahlen gibt.

(Beide Aussagen sind falsch.)

Hier sind einige der falschen Aussagen, die mir in den Sinn kamen und mich mindestens eine Augenbraue hochziehen ließen, als ich zum ersten Mal merkte, dass sie nicht wahr waren.


Jede lineare Funktion zwischen zwei Vektorräumen ist stetig.

Wahr nur, solange der Bereich endlichdimensional ist. Ist dies nicht der Fall, dann existiert eine lineare Funktion, die nicht stetig ist – an keinem Punkt!


Die Menge der reellen Zahlen kann auf keinen Fall (vollständig) so geordnet werden, dass jede nichtleere Menge darin ein kleinstes Element hat.

Falsch, wenn nach dem Wohlordnungssatz eine Auswahl angenommen wird.


Q ist nicht zählbar.

Ich bin immer noch versucht, es manchmal zu glauben ...


Wenn die Ableitung einer stetigen Reell-zu-Reell-Funktion fast überall existiert und (wo immer sie existiert) fast überall verschwindet, dann muss die Funktion konstant sein.

FALSCH. Tatsächlich gibt es eine Funktion, die die Prämisse erfüllt, und sie ist strikt [ sic! ] zunehmend!


Jede kompakte Menge ist abgeschlossen.

Der Name „kompakt“ würde dies suggerieren, was aber nur in Hausdorff-Räumen gewährleistet werden kann.


Eine Menge ist genau dann kompakt, wenn jede Folge in ihr eine konvergente Teilfolge enthält.

Während es in metrischen Räumen wahr ist, ist es nicht nur in einigen allgemeineren topologischen Räumen falsch, sondern impliziert auch keine Bedingung die andere!

Das glaube ich total Q ist zählbar. Meinten Sie R ? Nehmen Sie als Nächstes an, dass die Ableitung überall oder nur fast überall existiert?
Nein, meinte ich Q . Ich meine, duh, Q ist zählbar, aber ich brauche immer einen Vertrauensvorschuss, um es zu glauben. Wenn ich die Begründungen aufzählen darf [ 0 , 1 ] , warum kann ich das nicht in aufsteigender Reihenfolge tun? (Das heißt, so dass Q 1 < Q 2 < , Wo { Q N } N N = [ 0 , 1 ] Q ). Was das andere „Paradoxon“ betrifft, meinte ich, „existiert und verschwindet“ ist eine Schnittmenge von Ereignissen, deren Komplement das Maß Null hat. Differenzierbarkeit wird nicht überall vorausgesetzt. Ich werde es klarer machen, danke, dass Sie mir die Mehrdeutigkeit klar gemacht haben.
Das habe ich mir eingeredet Z 2 ist zählbar; offensichtlich ist es möglich, einen Algorithmus zu entwickeln, um sie zu zählen. Und ich habe mich davon überzeugt, dass dies das bedeutet Q ist zählbar, weil Q ist in Elementpaare geordnet Z 2 , also sind "weniger" Elemente drin Q als Z 2 (Denken Sie an grafische Elemente, Q hinterlässt Löcher, weil 2 4 = 1 2 ), So Q ist definitiv nicht überabzählbar unendlich.
@Quincunx Du hast vollkommen Recht. Es klingt einfach intuitiv sehr seltsam für mich. Sag das zum Beispiel F : R R ist eine Funktion. Angenommen, das F ist stetig, außer an einer zählbaren Menge von Punkten. Sie könnten versucht sein zu denken, dass diese Funktion „fast brav“ ist, oder? Es stellt sich heraus, dass es eine Funktion gibt, die genau an allen rationalen Punkten unstetig ist. An dieser Stelle würde ich schlussfolgern, dass die Funktion gar nicht „fast brav“ ist, sondern eher pathologisch. Immerhin ist die Menge der Diskontinuitäten von F ist dicht!
Ich denke, das einzige Merkmal von Q Was mich aus dem Gleichgewicht bringt, ist, dass es sich für mich irgendwie nicht „richtig“ anfühlt, wenn eine zählbare Menge in einer unzählbaren dicht ist.

Die Hauptvermutung besagt, dass es auf einer Mannigfaltigkeit im Wesentlichen nur eine PL-Struktur gibt. Genauer gesagt besagt es, dass zwei beliebige Triangulationen eine gemeinsame Unterteilung haben. Der Grund, warum dies "offensichtlich wahr" erscheint, ist, dass Sie beide Triangulationen nehmen und übereinander legen können, indem Sie die Mannigfaltigkeit in eine Reihe von Zellen unterteilen und dann die baryzentrische Unterteilung nehmen, um eine Triangulation zu erhalten. Es stellt sich heraus, dass dies falsch ist und man einige ziemlich subtile Invarianten braucht, um es zu entdecken. Das Problem mit dem Argument, das ich gegeben habe, ist, dass eine Triangulation in Bezug auf die andere sehr wild sein könnte (fraktisch wackelig), so dass ihre Vereinigung die Mannigfaltigkeit nicht in eine schöne Ansammlung von Zellen unterteilt.

Für welche Arten von Verteilern versagt es?
@MJD Soweit ich mich erinnere, gibt es eine Invariante der Grundgruppe (als Whitehead-Gruppe bezeichnet), die in Bezug auf Matrizen über dem Gruppenring definiert ist, und diese muss ungleich Null sein, damit die Mannigfaltigkeit die Chance hat, ein Gegenbeispiel zu sein. Dies schließt viele grundlegende Gruppen aus, einschließlich der trivialen Gruppe und Z . Es ist schon eine Weile her, dass ich darüber nachgedacht habe.

Steins Paradoxon ist für mich der verwirrendste mathematische Begriff, den ich je gekannt habe (obwohl ich kein Mathematiker bin), hauptsächlich weil es kein mathematisches "Artefakt" ist, aber seine Nicht-Intuition sehr greifbare Fehlerfolgen mit sich bringt.

Satz: (falsch)

Man kann nichts Besseres tun als eine gewöhnliche Entscheidungsregel zum Schätzen des Mittelwerts einer multivariaten Gaußschen Verteilung unter dem mittleren quadratischen Fehler.

Mit anderen Worten können tatsächlich völlig unabhängige Phänomene für einen geringeren gemeinsamen Schätzfehler kombiniert werden.

Was ist damit:

R Und R 2 sind nicht isomorph (wie abelsche Gruppen mit Addition).

Es fällt unter die Kategorie „Nehmen wir die Hamel-Basis von R ...", aber ich mag es sehr.

"Eine Folge von Zahlen, in der jede Zahl größer als die vorherige ist, wird immer irgendwann einen bestimmten Wert L überschreiten."

Ihre Antwort entspricht im Wesentlichen dieser .
Ahh, guter Fang, aber ich denke, meiner ist einfacher auszudrücken.

Ich möchte wirklich, dass Folgendes wahr ist:

Satz: Sei S eine Teilmenge eines Vektorraums. Wenn S ist paarweise linear unabhängig (d.h. jede { v , w } S linear unabhängig ist) dann S ist linear unabhängig.

Und doch ist es falsch. Zum Beispiel,

{ v , w , v + w }
Wenn S hatte nur zwei Elemente, dann gewinnen wir standardmäßig. Auf jeden Fall neigen die Studenten dazu, dies zu glauben. Ich meine, es ist lineare Algebra, das Prinzip der Überlagerung sollte gelten, oder? Etwas ist die Summe seiner Teile, lineare Unabhängigkeit erzeugt lineare Unabhängigkeit ... sehr verführerisch, sehr falsch.

Gibt es einen Vektorraum, in dem alle Vektorpaare linear unabhängig sind? Vielleicht meinst du eine Vektorraumbasis? Oder ist es üblich, den Vektorraum und eine Basis für einen Vektorraum zu verschmelzen?
@DanielV, nicht meine Bedeutung, ich werde umformulieren, ich meinte das, wenn jedes Paar von Vektoren drin ist S ist linear unabhängig ... danke für den Kommentar.
Ich mag Ihr Beispiel, ich wollte nur sichergehen, dass ich es nicht falsch interpretiere. Wenn ich mich richtig erinnere, sehe ich, dass sich "Vektorraum" auf die Spanne von Basisvektoren bezieht, sodass zwei verschiedene Basisvektorsätze denselben Vektorraum haben können. Der Klarheit halber hätte ich wahrscheinlich nur gesagt: "Wenn jedes Vektorpaar in S linear unabhängig ist, dann ist S eine Menge linear unabhängiger Vektoren". Entschuldigen Sie die Kritik, aber ich glaube, dass selbst eine grafische Beschreibung dieser Behauptung einige Schüler überraschen könnte, daher denke ich, dass dies ein gutes Beispiel ist.

Eines der ersten Male, bei dem ich dabei erwischt wurde, dass ich mich bei etwas so Offensichtlichem geirrt habe, war zu glauben:

abs(x) ist nie gleich -x

Natürlich abs(x)ist wie für definiert-xx < 0

Ein verwandtes Nicht-Theorem, das meine Studenten glauben wollen, ist das A 2 = A . (Andererseits habe ich Studenten, die das glauben wollen A + B = A + B .)
@TobyBartels, je nachdem, wie Sie Dinge definieren A 2 = A könnte teilweise stimmen. Ist A 2 die (mehrwertige) Umkehrung von X X 2 bei A 2 , oder ist es die eindeutige positive Zahl B so dass B 2 = A 2 ? Die Leute (für meinen Geschmack ärgerlich) sprechen davon in beide Richtungen.
@CharlesHudgins: Das stimmt, obwohl es etwas schwierig ist, = mit mehrwertigen Operationen zu verwenden.
@TobyBartels Deshalb bin ich, pädagogisch gesehen, dafür, Schülern nicht das Lösen beizubringen X 2 = 9 durch „Ziehen der Quadratwurzel aus beiden Seiten“ und anschließendes Schlagen auf a ± . 1) Die Schüler werden das immer vergessen ± . 2) Es erweckt den falschen Eindruck, dass Sie nicht-injektive Funktionen einfach "rückgängig machen" können. Stattdessen sollten sie neu angeordnet werden X 2 9 = 0 und faktorisieren und dann lösen, damit sie verstehen, warum es zwei Antworten gibt. Das heißt, ich werde nicht die Luft anhalten. Pädagogische Veränderungen brauchen ewig, um sich in den Mathelehrplan einzuarbeiten.

Ich denke , dies wird in keiner der anderen Antworten behandelt (obwohl es sicherlich viele davon gibt). Das Simpson-Paradoxon ist nah, aber ich denke, das ist anders und etwas einfacher zu verstehen:

Wenn X positiv korreliert mit Y , Und Y positiv korreliert mit Z , Dann X positiv korreliert mit Z .

Mit anderen Worten, positive Korrelation ist transitiv. Ich denke, es ist ziemlich intuitiv, aber falsch.

Es ist jedoch wahr, dass, wenn beides X Und Z sind positiv und perfekt korreliert mit Y , Dann X positiv und perfekt korreliert mit Z .

"Offensichtlich"

( X j ) z = X j z
für X , j , z C so dass gegebene Ausdrücke definiert sind.

Jemand muss dies den Autoren von Lehrbüchern zur Verbesserung der Elementaralgebra auf College-Niveau sagen. Diese besagen konsequent, dass dies als Satz gilt, wenn x reell und y und z rational sind.

Etwas, zu dem ich mich in meiner mathematischen Unreife (die leider immer noch existiert) verführen ließ:

Nehme an, dass P N sind eine Familie von Anweisungen, die von indiziert werden N N und wir können Bedeutung zuordnen P . Dann wenn P N gilt für alle N N , Dann P stimmt auch.

Das hängt alles davon ab, wie genau Sie die "Bedeutung" von definieren P .
„Angenommen, wir könnten dem eine Bedeutung zuweisen P ." Ich gehe davon aus, dass wir das können.
In diesem Fall nenne ich ein Beispiel, wo die P ist "richtig" definiert, ist aber eigentlich falsch.
Das Beispiel, das ich im Sinn hatte, als ich das schrieb, war P N : die Summe von N differenzierbare Funktionen ist differenzierbar; P : Unendliche Summe differenzierbarer Funktionen ist differenzierbar.
Wie wäre es mit der Aussage, dass 1/n > 0 gilt, was für jede ganze Zahl n gilt, aber im Unendlichen versagt?
@JónÁskellÞorbjarnarson Mark würde das behaupten 1 / > 0 ist nicht aussagekräftig, weil 1 / ist nicht definiert. Ein einfacheres Beispiel ist nur eine Folge von rationalen Argumenten, die zu einem irrationalen konvergieren.
@Jp: Wir können klar Bedeutung zuordnen 1 / . Es ist sogar nützlich. Es ist nur "nicht definiert" in dem Sinne, dass eine gewöhnliche Summe von unendlich vielen Dingen nicht definiert ist, oder das für eine Familie P N mit N N , P ist nicht definiert.
@Hurkyl Ich stimme zu und habe versucht, von Marks Position aus zu argumentieren.
Das ist das Leibnizsche Gesetz der Kontinuität, richtig?
@TannerSwett Nach einem schnellen Google glaube ich, dass es so ist.

Der Satz von Cauchy impliziert, dass:

Wenn man ein physikalisches Modell eines konvexen Polyeders herstellt, indem man starre Platten für jede der Polyederflächen mit flexiblen Scharnieren entlang der Polyederkanten miteinander verbindet, dann bildet dieses Ensemble von Platten und Scharnieren notwendigerweise eine starre Struktur.

Es gibt jedoch Gegenbeispiele, wenn Sie ein allgemeines Polyeder (nicht konvex) zulassen.

EDIT: Das Gegenbeispiel, das ich im Sinn hatte, ist falsch; Ich habe eine andere Frage gestellt , um zu versuchen, die Angelegenheit auf die eine oder andere Weise zu klären. Unabhängig davon denke ich, dass es sicher ist zu sagen, dass dies nicht mehr offensichtlich ist! Aber ich werde meine Antwort entsprechend aktualisieren (oder löschen), sobald ich etwas mehr Klarheit habe.

Hier ist ein topologisches Beispiel, dessen Verfälschung einige Überlegung erfordert: Grob gesagt „gibt es für jede sich nicht schneidende Kurve zwischen zwei gegenüberliegenden Ecken eines Quadrats eine Kurve zwischen den beiden anderen Ecken, die es nur einmal schneidet“. Formal:

Lassen F : [ 0 , 1 ] [ 0 , 1 ] 2 sei eine sich nicht selbst schneidende Kurve mit F ( 0 ) = ( 0 , 0 ) , F ( 1 ) = ( 1 , 1 ) , Und F ( T ) ( 0 , 1 ) 2 für T ( 0 , 1 ) . Dann gibt es eine sich nicht selbst schneidende Kurve G : [ 0 , 1 ] [ 0 , 1 ] 2 mit G ( 0 ) = ( 1 , 0 ) , G ( 1 ) = ( 0 , 1 ) , Und G ( T ) ( 0 , 1 ) 2 für T ( 0 , 1 ) so dass es einzigartige gibt T 0 Und T 1 mit F ( T 0 ) = G ( T 1 ) .

Dies scheint (zumindest für mich) auf den ersten Blick offensichtlich, und sogar auf den zweiten Blick legt das Beispiel des Jordan-Kurven-Theorems nahe, dass es wahr sein sollte; Schließlich erhalten wir durch die JCT eine „linke Seite“ und eine „rechte Seite“ unserer Kurve, und bedeutet das Theorem von Schoenflies nicht, dass wir in der Lage sein sollten, eine inverse Abbildung unserer Kurve auf den Kreis zu finden? Aber es ist falsch; es gibt kurven F ( ) die von keiner Kurve nur einmal geschnitten werden kann G ( ) . Ein Gegenbeispiel zu finden, ist eine schöne Übung ...

Bist du dir sicher? Ich glaube, ich kann beweisen, dass ein solches g immer existiert.
@studiosus Ich würde gerne deinen Beweis sehen! Ich war ziemlich zuversichtlich in mein Beispiel, aber wenn ich weiter darüber nachdenke, bin ich tatsächlich bereit zu glauben, dass es falsifiziert werden kann. Ich habe vielleicht nicht genügend entartet in Betracht gezogen G ( ) mit meinem kanonisch entartet zu gehen F ( ) ...
In den Kommentaren ist kein Platz für einen Beweis, da es lang ist. Stellen Sie eine separate Frage.
Hm, ich würde sagen, es kann als Folge des Jordan-Schoenflies-Theorems gezeigt werden (für das ich keinen Beweis kenne).
Ich würde w / @Alexey zustimmen. Fügen Sie eine Kurve von hinzu ( 1 , 1 ) Zu ( 0 , 0 ) erweitern F zu einer geschlossenen Kurve F ~ Das ( 0 , 1 ) ist drinnen u ( 1 , 0 ) ist draußen. Finden Sie einen Homöomorphismus Φ : R 2 R 2 der Ebene, die abbildet F ~ zum Einheitskreis (nach dem Jordan-Schoenflies-Theorem). Zeichnen Sie ein gerades Segment, das die Bilder von verbindet ( 0 , 1 ) Und ( 1 , 0 ) (dies kreuzt den Einheitskreis genau einmal). Das Pre-Image dieses Segments (also sein Image unter Φ 1 ) ist die gewünschte Kurve G . (Rechts?)
@mjqxxxx: Nein, das funktioniert nicht: Das gerade Liniensegment ist möglicherweise nicht enthalten Φ ( [ 0 , 1 ] 2 ) (da Sie nicht wissen, wie das Bild der restlichen Begrenzung des Einheitsquadrats aussieht). Man braucht mehr Arbeit, um einen echten Beweis zu bekommen.
@studiosus: Du hast Recht! Das ist mir nicht aufgefallen. Und es gibt keinen Grund, das Bild des Innenraums der Einheit unter Platz zu nehmen Φ soll auf jeden fall schön sein. Vielleicht ist das hier die Falte.
Das ist ein kleiner Stich ins Blaue, aber kann es überhaupt ein solches g geben, wenn f eine raumfüllende Kurve ist?
@ChristopherCreutzig: Beide Karten f und g sind 1-1, können also nicht raumfüllend sein.
Hier ist ein stärkerer (begradigender) Satz: Let S eine Fläche mit einer festen Triangulation sein (z. B. eine triangulierte Ebene) und H : G S eine topologische Einbettung eines endlichen Graphen. Dann H ist isotopisch zu einer stückweise linearen Einbettung H ' : G S . Dies reduziert Probleme der obigen Art auf stückweise lineare, die leicht lösbar sind. Man kann jedoch eine direktere Lösung geben, indem man nur Schoenflies und ein wenig algebraische Topologie verwendet.
Ja, meine Gegenbeispiele scheitern tatsächlich alle an der Kontinuität, ich hatte nicht genau genug darauf geachtet (sie hatten „Akkumulationslinien“, den Sinuskurvenstil des Topologen, aber das bricht natürlich die Annahme). Ich werde diesen nicht falschen Satz gleich löschen und mein Beispiel als richtige Frage stellen, da der Beweis für mich wirklich interessant klingt!
@studiosus (Sie finden diese Frage unter math.stackexchange.com/questions/821025/… wenn Sie die Frage dort richtig beantworten möchten!)

Das Folgende ist offensichtlich falsch, aber es ist tatsächlich wahr, wie im Wikipedia-Artikel über Vitali-Sets gezeigt wird .

Es existiert eine zählbare Sammlung { v N } von Teilmengen des Einheitskreises, so dass:

  1. Irgendwelche zwei unterschiedlich v N sind disjunkt.
  2. Beliebig v N kann durch eine Drehung von jedem anderen erhalten werden.
  3. Die Vereinigung aller v N ist der ganze Kreis.

All die v N muss nach Eigenschaft zwei die gleiche "Größe" haben (für jede vernünftige Definition von "Größe"), aber wenn die obige Tatsache wahr wäre, wäre die Summe ihrer (gleichen) Größen die Größe des Kreises (positiv, aber endlich). Aber wenn die Größe Null war, sollte die Summe Null sein, und wenn die Größe positiv war, sollte die Summe unendlich sein.

Eine Folge davon ist, dass Folgendes falsch ist (obwohl wir es alle gerne wahr hätten):

Es existiert eine Funktion μ dass, gegeben eine begrenzte Teilmenge von R , sagt Ihnen seine "Größe". Genau:

  1. Wenn A R ist dann begrenzt μ ( A ) [ 0 , [ .
  2. Wenn { A N } N N ist eine Folge von beschränkten disjunkten Teilmengen von R (das ist, A N A M = wann immer N M ) mit beschränkter Vereinigung (das heißt, A N ist beschränkt), dann μ ( A N ) = μ ( A N ) .
  3. Wenn A ist begrenzt, X eine reelle Zahl ist, und wir definieren A + X = { A + X : A A } , Dann μ ( A + X ) = μ ( A ) .
  4. μ ( [ 0 , 1 ] ) = 1

Tatsächlich wird die erste Tatsache umgeschrieben, indem der Kreis durch das halboffene Intervall ausgetauscht wird [ 0 , 1 [ und Austausch von Rotationen gegen zyklische Verschiebungen "mod 1 “, das merken wir, wenn μ dann die ersten drei Bedingungen oben erfüllt μ ( A ) = 0 für alle A .

Wenn Sie eine endliche Anzahl von Mengen betrachten v N , und nehmen Sie eine Grenze, die plausibel erscheint, nicht ganz offensichtlich falsch.

Das Geburtstagsparadoxon

Wenn 30 Personen zufällig ausgewählt werden und sie unabhängig voneinander (identisch) gleichmäßig über das Kalenderjahr verteilt Geburtstage haben, dann ist die Wahrscheinlichkeit, dass zwei (oder mehr) von ihnen denselben Geburtstag haben, ungefähr gleich 1 12 .

Vor 1955 „wusste“ das jeder, die n-te Dezimalstelle von zu kennen π (und für alle anderen irrationalen) war es notwendig, die vorherigen Ziffern zu kennen. Ein Genie wie Archimedes (" In Archimedes Kopf steckte mehr Vorstellungskraft als in Homers Kopf ": Voltaire) "wusste" dies sehr gut, wie die Geschichte zeigt. Die Bailey-Borwein-Plouffe-Formel (BBP-Formel) endete jedoch mit diesem seit Jahrhunderten heiligen „Wissen“, und jetzt ist es möglich, zum Beispiel die 33-te Ziffer zu kennen, ohne die Präzedenzfälle zu kennen.

Was die intuitive Wahrnehmung betrifft, so ist es falsch, dass eine stetige numerische Funktion zumindest in einem Punkt ableitbar sein muss; es ist auch falsch, dass ein kleines Quadrat keine Kurve von unendlicher Länge enthalten kann.

Die BPP-Formel für Pi gibt keine Dezimalziffern, sondern Hexadezimalziffern an.
Ich denke, "1955" muss hier ein Fehler sein.
Die Idee, dass „ein kleines Quadrat keine Kurve von unendlicher Länge enthalten kann“, wird weniger „offensichtlich“, wenn Menschen erkennen, dass jede winzige Zelle in ihrem Körper sehr lange DNA-Stränge enthält. (Natürlich ist die DNA nicht ganz unendlich lang und die Zelle ist dreidimensional, also ist dies kein Widerspruch gegen die "offensichtliche" Idee, aber es scheint die Offensichtlichkeit zu ruinieren.)

Hier sind einige "offensichtliche" Aussagen, auf die Richards Paradoxon zutreffen kann:

  1. Für ein gegebenes Prädikat P , es existiert eine Menge S von X wofür P ( X ) ist wahr. (Russells Paradoxon)
  2. Die Menge der ganzen Zahlen und die Menge der reellen Zahlen haben dieselbe unendliche Größe. (Cantors diagonales Argument.)
  3. Es gibt eine Formalisierung der Arithmetik, in der alle wahren Aussagen genau die sind, die beweisbar sind. (Satz von Gödel)
  4. Es gibt ein Computerprogramm (Turing-Maschine), das effektiv feststellen kann, ob ein anderes Computerprogramm nicht anhält. (Halteproblem)
@MJD Ja und ich habe +1 gegeben :) Dein Harfen ist gut begründet.
Ich denke, Nr. 4 ist nicht nur nicht offensichtlich wahr, sondern offensichtlich falsch, denn wenn es wahr wäre, müssten wir unsere Computerprogramme nicht wirklich ausführen, um herauszufinden, was sie getan haben, sondern wir könnten dieses hypothetische Verfahren verwenden, um es uns zu sagen , ohne sie auszuführen. Aber welchen Sinn hätte es dann, die Programme überhaupt laufen zu lassen? Das Universum funktioniert einfach nicht so; Wenn Sie herausfinden wollen, was passiert, müssen Sie es zumindest manchmal einfach ausprobieren und sehen.
Auch #2 wird bereits in meiner Antwort erwähnt.
@MJD Entschuldigung, ich habe gerade einen Tippfehler in # 4 korrigiert, in dem ich am Ende des Satzes "halt" durch "nicht angehalten" ersetzt habe.
Es spielt keine Rolle; Mein Kommentar ist so oder so ähnlich. Ich habe einen Blogbeitrag verfasst, der behauptet, dass der Satz von Rice der offensichtlichste Satz in der Mathematik ist und die Unentscheidbarkeit des Halteproblems ein Sonderfall davon ist.
@MJD Ich sehe die Essenz deines Standpunkts nicht. Ein Teil des Anhalteproblems ist, wenn ein Programm nicht anhält, woher wissen wir das? Wir können es nicht einfach „ausprobieren und sehen“ – wie lange warten wir, bevor wir entscheiden, dass es nicht aufhört? Das ist das wesentliche Problem. Diese Art von Aussage mag zu Beginn des 20. Jahrhunderts "offensichtlicher" gewesen sein, als Automatisierung der Name des Spiels zu sein schien, was sich auch in den Zielen von Russell und Whitehead bei der Formalisierung der Mathematik widerspiegelte. Es ist großartig, dass es für Sie jetzt offensichtlich falsch ist, aber ich würde sagen, dass wir aufgrund solcher Ergebnisse jetzt so aufgeklärt sind.
Lassen Sie es mich anders ausdrücken. Wenn das Haltetheorem falsch wäre, gäbe es eine Methode, um ziemlich schnell zu sagen, was das Ergebnis einer Berechnung sein würde, selbst wenn dieses Ergebnis in ferner oder unendlicher Zukunft liegt. Aber wir wissen , dass es unmöglich ist, die ferne Zukunft vorherzusagen; es ist in der gleichen Fantasy-Kategorie wie Kobolde und Einhörner. Warum sollten wir uns also wundern, dass es tatsächlich keine Möglichkeit gibt, jetzt zu sagen, was eine Turing-Maschine in der Zukunft tun wird? Sie behaupten, es sei offensichtlich (aber falsch), dass wir die unendlich ferne Zukunft korrekt vorhersagen könnten. Warum?
Hier geht es nicht um allgemeine Zukunftsprognosen; mit Zeit hat dies nur insofern zu tun, als zählbare Unendlichkeit auf diskrete Turing-Maschinenschritte abgebildet werden kann, die in der Zeit stattfinden. ZB eine Turing-Maschine, die anhält, wenn sie etwas findet A , B , C , N Z + : A N + 2 + B N + 2 = C N + 2 wir wissen, dass es dank Wiles' Beweis nicht anhält. Wenn das Haltetheorem falsch wäre, dann würde eine „Theorem-beweisende Maschine“, die auf einer Formalisierung irgendeines Bereichs der Mathematik basiert, schließlich alle wahren Aussagen ausspucken. Dies war Russell, Whitehead und sogar Hilbert bis in die 1930er Jahre „offensichtlich“, dank Gödel, Turing et al.
@MJD Wenn der Haltesatz falsch wäre, bedeutet dies nicht, dass wir das Ergebnis einer Berechnung feststellen könnten. Es würde lediglich implizieren, dass ein Algorithmus existiert. Es besteht noch eine beträchtliche Lücke zwischen der Existenz eines solchen Algorithmus U , und sowohl (1) unsere körperliche Fähigkeit zu laufen U (Es kann mehr Speicher erfordern als die Anzahl bekannter Atome) und (2) noch wichtiger unser Wissen , dass ein bestimmter Algorithmus tatsächlich existiert U . (1) und (2) folgen nicht automatisch aus der Falschheit des Haltesatzes. Die Gründe dafür, dass es wahr ist, sind subtiler, als Sie es darstellen.
@MJD Ich stimme Matt zu. Selbst wenn das Haltetheorem falsch wäre, bedeutet das nicht, dass der Algorithmus überhaupt praktikabel wäre, und natürlich verlangt die Vernunft, dass er völlig unpraktisch ist. Wie Matt betont, kann eine Theorembeweismaschine jede wahre Aussage ausspucken, wenn der Haltesatz falsch war; da es wahr ist, wird ein solches Programm nur beweisbare Aussagen ausspucken und Ihnen nicht sagen können, dass ein gegebenes Theorem nicht beweisbar ist, es sei denn, Sie warten ewig. Aber selbst das klingt großartig - warum lassen wir nicht Theorem-Beweismaschinen laufen, um alle beweisbaren Aussagen auszuspucken und Mathematiker aus dem Geschäft zu drängen?

Es gibt viele Beispiele in der (extremen) Graphentheorie, wo ein offensichtliches Argument zeigt, dass eine Aussage wahr ist, außer dass es eine Reihe kleiner Gegenbeispiele gibt, die leicht zu übersehen sind.

Betrachten Sie die folgende Aussage: Let G sei ein Graph mit N Ecken und die größte Anzahl von Kanten unter der Bedingung, dass G enthält kein Paar disjunkter Kanten (d. h K 2 + K 2 ). Dann G ist ein Stern (z K 1 , N 1 ).

Das ist offensichtlich wahr, wenn man einen Moment darüber nachdenkt. Aber für N = 3 , ist eine bessere Lösung zu nehmen G = C 3 . Und für N = 4 , nehmen C 3 plus ein isolierter Scheitelpunkt ist genauso gut wie das Nehmen K 1 , 3 .

Eine lineare Ordnung kann eindeutig (bis auf Isomorphie ) aus der Menge der Ordnungstypen ihrer eigentlichen Anfangssegmente rekonstruiert werden .


Update: Selbst wenn wir die Kardinalität der linearen Ordnung kennen und wissen, dass sie kein maximales Element hat, gilt dieser "Satz" immer noch nicht.

Alle Unendlichkeiten sind gleich groß.

Aber der Satz von Cantor zeigt etwas anderes.

Eine Untergruppe einer endlich erzeugten Gruppe darf nicht endlich erzeugt werden und es gibt bis auf Isomorphie höchstens zwei P Q Gruppen, wo P Und Q sind prim.

Um Ihr Beispiel zu stärken, P   P R ich M e 1 / P weicht ab.

Die Kontinuumshypothese scheint auch in ZFC eine Antwort zu haben, was nicht der Fall ist.

Auf einer anderen Seite dachten Mathematiker, dass zyklotomische Felder "offensichtlich den einzigartigen Faktorisierungssatz erfüllen", was zu einigen falschen Beweisversuchen für Fermats letzten Satz führte.

Als nächstes könnte man denken, dass die "Winkeldreiteilung" möglich ist oder dass jede Menge analytischer Funktionen { F a } , so dass für jeden z C , der Satz { F a ( z ) } zählbar ist, muss selbst zählbar sein.

Dies sind nur einige zufällige Beispiele, die mir in den Sinn gekommen sind, und da der Begriff „offensichtlich“ subjektiv ist, können Sie den Punkten auf meiner Liste sehr wohl widersprechen. Ich denke, es hängt stark von Ihrem mathematischen Hintergrund ab.

Ich bin nicht der Meinung, dass irgendetwas an der Kontinuumshypothese intuitiv offensichtlich ist, und ich finde noch weniger, was intuitiv offensichtlich ist, was in ZFC bewiesen werden kann oder nicht, was so komplex ist, dass man wirklich hart studieren muss, nur um zu verstehen, was die Axiome sind Sprichwort.

Wenn Sie anfangen, über die Steifigkeit dünner Schalen nachzudenken R 3 , stoßen Sie schnell auf eine Reihe kontraintuitiver Ergebnisse.

Zum Beispiel ist es offensichtlich, dass eine Kugelschale ( C 2 ) starr, und das stimmt tatsächlich. Eine glatte, geschlossene, kompakte Fläche mit überall positiver Gaußscher Krümmung ist ebenfalls starr. Man könnte sich vorstellen, diese Ergebnisse zu verallgemeinern

  • Jede geschlossene Oberfläche;
  • Jede geschlossene Fläche mit positiver Gaußscher Krümmung überall, aber an endlich vielen Punkten;
  • Jede Fläche mit Rand mit überall positiver Gaußscher Krümmung;

und all dies ist falsch.

Darüber hinaus ist es, nachdem man über Reflexionen nachgedacht oder einen Tischtennisball angestupst und angestoßen hat, intuitiv offensichtlich, dass eine kugelförmige Hülle dies nicht ist C 0 starr. Aber Sie können keinen wirklichen Unterschied zwischen a C 1 und ein C 2 Verformung der Kugel, so sicher ist die Kugel C 1 starr? Weit gefehlt – angesichts einer beliebigen geschlossenen Fläche, die topologisch eine Kugel ist, und einer Entfernung ϵ , es ist möglich zu C 1 - Eine Kugel isometrisch einbetten ϵ -nah an der Zielfläche!

Das Folgende ist ein sehr bekanntes Beispiel, wenn auch wahrscheinlich etwas außerhalb der Welt der Mathematik, eher der Physik. Sehr viele Menschen würden „intuitiv“ Folgendes für wahr halten:

The heavier the object, the faster it falls down.

Tatsächlich besagt die Geschichte, dass dies allgemein bekannt sein sollte, bis Galileo Galilei es widerlegte (wie die Geschichte besagt, indem er zwei Kugeln des Turms in Pisa fallen ließ, was jedoch nie geschah).

Einer der ersten Physikkurse, die viele Leute haben (ich spreche hier von der Grundschule), zielt darauf ab, zu zeigen, dass dieses Theorem falsch ist, und tatsächlich fällt alles mit der gleichen Beschleunigung (ohne Berücksichtigung des Luftwiderstands), unabhängig vom Gewicht.

Alles fällt mit der gleichen Beschleunigung, nicht mit der gleichen Geschwindigkeit ... großer Unterschied.
Die Behauptung von Aristoteles war tatsächlich, dass Objekte mit einer Geschwindigkeit fallen, die proportional zu ihrem Gewicht ist , was viel spezifischer ist und so viel weniger offensichtlich erscheint. Es ist unglaublich, dass die westliche Zivilisation diesen Whopper so lange geschluckt hat.
@MJD Allgemein neigen Menschen dazu, Dinge eher nach Popularität als nach Beweisen oder Erfahrungen zu glauben. Dies gilt heute wie in der Vergangenheit, und Studien zeigen, dass (kontraintuitiv) gut gebildete Menschen besser darin sind als schlecht gebildete Menschen.
Ich mache mich in meinem Unterricht über diese Idee lustig, wenn ich die Antworten mehrerer Schüler überprüft habe und die Modusantwort auswähle und verkünde: "Diese hier muss richtig sein, denn wie wir alle wissen, ist Demokratie der Weg zu aller Wahrheit!"
Aber ein Gasballon ist leichter (relativ zu seinem Volumen) als zB ein Stein und er steigt sogar, anstatt herunterzufallen.

Es ist nicht gerade ein Theorem, aber es täuscht jeden Mathe-Neuling:

e = lim N ( 1 + 1 N ) N

( 1 + 1 / ) Ist 1 , offensichtlich. Und 1 hoch ist offensichtlich immer noch 1.

Nein, es ist 2,718 ...

Ich glaube, das Poster hat dieses Beispiel in der Frage gegeben, aber es ist ein gutes.
Nö, 1 ist eine unbestimmte Form.

Hier ist eine Behauptung, von der ein Durchschnittsmensch behaupten würde, dass sie wahr ist, wenn sie in Laienform ausgedrückt wird.

Jede Teilmenge reeller Zahlen hat ein Maß.

Wie kann das falsch sein, wenn Sie eine Region markieren, sagen wir in zwei Dimensionen, natürlich hat sie eine Fläche?! Sofern Sie nicht irgendwann eine Vitali-Menge konstruiert haben, neigen wir dazu zu glauben, dass das Konzept von Länge/Fläche/Volumen sich auf alle möglichen Teilmengen erstrecken sollte.

Hier ist eine weitere solche falsche Behauptung.

Axiom der Bestimmtheit

Wenn wir ein Endlosspiel für zwei Spieler spielen, in dem wir eine reelle Zahl erzeugen [ 0 , 1 ] indem wir abwechselnd Dezimalziffern wählen und einer von uns versucht, die resultierende Zahl in einem vorbestimmten Auszahlungssatz zu landen, der uns beiden bekannt ist, und der andere versucht, dies zu vermeiden, wie könnte es sein, dass es ein Spiel gibt, bei dem keines von beidem vorkommt uns haben eine Gewinnstrategie? Wir haben beide vollständige Informationen über den Auszahlungssatz, welche Zahlen zu vermeiden und welche Zahlen zu treffen sind, einer von uns sollte in der Lage sein, eine Strategie zu entwickeln. Nun, leider nein.

Diese beiden Aussagen widersprechen dem Axiom der Wahl, mit dem Sie die Gegenbeispiele konstruieren können, die sich nicht "nett benehmen".

Tatsache: Der letztere Satz impliziert den ersteren. (in ZF, mit der angenommen wird, dass AD konsistent ist).

Es erscheint etwas übertrieben, AD als „falsch“ zu bezeichnen. AC befindet sich auf keiner höheren erkenntnistheoretischen Ebene.
Wie würden Sie "jede Menge reeller Zahlen hat ein Maß" in Laiensprache ausdrücken, damit eine durchschnittliche Person behaupten würde, dass es wahr ist? Stellen Sie sicher, dass Sie die zählbare Additivität in die Formulierung Ihres Laien aufnehmen, da dies erforderlich ist, um die Aussage falsch zu machen.
@bof: Das Maßkonzept stammt aus unserer intuitiven Vorstellung von Länge/Fläche/Volumen. Ich bezweifle, dass ein durchschnittlicher Mensch dem widersprechen würde. Wählen Sie ein Einheitsquadrat aus und färben Sie eine Teilmenge davon ein. Es sollte eine gewisse "Fläche" haben, die mit Flächen von Grundformen vergleichbar ist. Oder fragen Sie dies, wenn ich einen Dartpfeil auf dieses Feld werfe, glauben Sie, dass es eine genau definierte Wahrscheinlichkeit gibt, dass ich den farbigen Bereich treffe? Was die zählbare Additivität betrifft, schließt unsere Intuition über "Fläche" diese nicht ein? Wenn ich Ihnen disjunkte Stücke gebe, die Sie mit Naturzeichen beschriften können, ist die Fläche der Vereinigung natürlich die Summe der Flächen?
@KevinCarlson Keine Dehnung, aber allgemeiner Gebrauch. „Falsch“ (ohne Qualifizierer, wie in „falsch in diesem oder jenem Modell“) ist eine Abkürzung für „falsch unter der Standardinterpretation“, was in diesem Fall unter dem Standardsatz von Axiomen bedeutet, nämlich Z F C , einschließlich Wahl.

Kontraintuitives Beispiel

    D v F ( A ) = 0 , v , A F     kontinuierlich .

Von D v meinst du F ( v ) , wie in der v -te Ableitung?

Geometriebeweise informell durch Zeichnen von Figuren an der Tafel. Sie umgehen dann die Axiome der euklidischen Geometrie, Sie tun so, als müssten Sie sie nicht aufrufen, da die gezeichneten Zahlen ausreichend erscheinen. In der Schwerkraft der Erde ist die euklidische Geometrie jedoch nur eine Annäherung.

Unendlich viele Terme haben immer eine Summe gleich unendlich.

Meinst du unendlich viele Terme? Wenn nicht, was ist ein "unendlicher Begriff"?
Das sind unendlich viele Begriffe. :)
Hey, ich glaube nicht, dass das stimmt, weil eine geometrische Folge unendlich viele Terme hat, die sie immer noch konvergiert.

Jemand anderes erwähnte "es gibt mehr rationale Zahlen als ganze Zahlen". In ähnlicher Weise fiel es mir schwer, das zu akzeptieren

Es gibt mehr ganze Zahlen als reelle Zahlen zwischen 0 und 1

ist falsch. Ich meine, ich verstehe es jetzt, aber es erschien mir intuitiv sehr falsch, bevor ich mich mit transfiniten Zahlen befasste.

Ein weiteres Beispiel dafür, dass „offensichtlich“ nicht wahr ist, ist das Bus Waiting Time Paradox .

Wenn die mittlere Zeit zwischen zwei aufeinanderfolgenden Bussen, die an einem Busbahnhof ankommen, ist M , sollte man damit rechnen, dass die mittlere Wartezeit am Bahnhof bis zur Ankunft des nächsten Busses ebenfalls M ist. Aber das stimmt nicht; und abhängig von der jeweiligen Busankunftszeitverteilung müssen Sie eine Zeit warten M ' M

Für mich ein schönes Beispiel für alle "Beweise", die darauf hindeuten, dass es wahr ist

π ( X ) < li ( X )

bis Skewes das gezeigt hat π ( X ) li ( X ) wechselt unendlich oft das Vorzeichen

Die Frage verlangt offensichtliche, aber falsche Aussagen. Behaupten Sie das ernsthaft π ( X ) < li ( X ) Es ist offensichtlich?
schwer zu sagen, ob das offensichtlich sein soll... Was sind π ( X ) Und li ( X ) ?
π ( X ) ist die Primzahlzählfunktion , l ich ( X ) ist das logarithmische Integral. Ramanujan behauptete die Ungleichheit. Littlewood bewies 1914, dass sich das Vorzeichen unendlich oft ändert, und Skewes gab eine Obergrenze für die erste Änderung an.

Hier ist eine Sache, die allgemein für wahr gehalten wird, aber in vielen Fällen ziemlich falsch ist:

There is a notion of mathematics where we can say things are 
"actually true" or "actually false".  

Ein Beispiel für diesen Fehler: das OP. Andere Beispiele: die vielen Antworten.

Es gibt mehrere Gründe, warum dies falsch ist. Erstens haben wir in dem System, von dem die meisten Mathematiker ausgehen, wenn es nicht explizit ist, kein Standardmodell (wir haben keine Modelle innerhalb dieses Systems, weil jedes Modell das System konsistent zeigen würde, von dem wir wissen, dass wir es in diesem System nicht durch blah blah Godel blah zeigen können. Ich weiß, dass Sie keine Details wollen, sondern nur erklären, worauf ich hinaus will). Wahrheit und Falschheit sind semantisch – sie existieren in Modellen, und ohne eines erheben wir keine Behauptungen über Wahrheit oder Falschheit.

Aber auch Mathematik ist nicht "das System, das die meisten Mathematiker annehmen, wenn sie nicht explizit sind" - es ist Formalisierung im Allgemeinen. Es gibt viele Systeme, die ernsthaft von Mathematikern untersucht werden, die zahlreiche "kontraintuitive" Ableitungen machen. Zum Beispiel sind diese in verschiedenen Systemen alle offensichtlich und falsch:

  • Eine Aussage kann nicht gleichzeitig wahr und falsch sein. (In der parakonsistenten Logik können Aussagen sowohl wahr als auch falsch sein und das System kollabiert nicht zu trivial – tatsächlich argumentieren eine Reihe von Diatheisten, dass dies ein viel genaueres logisches System für das Denken in der realen Welt ist).
  • Es gibt unstetige Gesamtfunktionen. (In einer Reihe konstruktiver Systeme ist es nicht möglich, die Existenz unsteter Gesamtfunktionen zu beweisen. Einige sind sogar stark genug, um zu beweisen, dass alle Gesamtfunktionen stetig sind.)
  • Jede unendliche Menge A hat die gleiche Kardinalität wie AxA. (Dies gilt nicht unbedingt für Systeme ohne Wahlaxiom. Bekanntlich versuchte Tarski, sein Ergebnis zu dieser Implikation zu veröffentlichen, und wurde sowohl von Frechet als auch von Lebesgue abgelehnt. Frechet dachte, dass das Papier offensichtlich und bekannt sei und keinen mathematischen Wert habe. Lebesgue dachte, sowohl das Axiom der Wahl als auch die Implikation daraus seien falsch, daher hatte die Arbeit keinen mathematischen Wert.)

Ich erwähne diese Beispiele nur nicht als Antworten auf das OP, sondern nur, um meine eigentliche Antwort zu veranschaulichen, dass die Frage selbst eine äußerst häufige Annahme in der Mathematik demonstriert, die tatsächlich falsch ist.

BEARBEITEN

Dies ist ein Bereich, von dem ich denke, dass er oft ein Ort häufiger Missverständnisse ist, und die Diskussion in den Kommentaren macht deutlich, dass ich näher darauf eingehen sollte. Die moderne Mathematik trennt die Bereiche, über die wir Aussagen machen, in Syntax und Semantik.

Syntax

Die Syntax ist die Theorie - die formale Sprache, Axiome, die als Sätze in der formalen Sprache spezifiziert sind, und einige metalogische Schlußregeln. In der Syntax sprechen wir von Sätzen, Aussagen, Termen, Ableitungen und Beweisen. Es ist ein Ort der Symbolmanipulation.

Semantik

Die Semantik ist das Modell – es ist die Bedeutung, die wir den Aussagen der Theorie zuschreiben. Eine Interpretation einer Theorie ist ein Modell, das jeder Formel der Theorie einen Bedeutungswert zuweist – typischerweise Wahrheit. Wahrheit ist semantisch und spezifisch für ein Modell.

Das Problem"

Ein Modell ist eine konsistente Interpretation der Wahrheitsbedeutung einer Theorie. Wenn eine Theorie ein Modell hat, hat sich fast trivialerweise gezeigt, dass sie widerspruchsfrei ist. Aber ... es ist bekannt, dass eine Theorie, die stark genug ist, um die Gödel-Diagonalisierung auszudrücken, niemals ihre eigene Konsistenz beweisen kann. Für diese Theorien werden wir nie ein Modell haben und können keine Aussage über die Bedeutung irgendeiner Formel treffen.

In diesen Theorien ist es falsch, von Wahrheit oder Falschheit zu sprechen. Wir haben kein Modell, das dem eine Bedeutung gibt. Wir werden nie ein Modell haben.

Das ist nicht wirklich ein Problem. Mathematiker hatten jahrhundertelang Herleitung und Wahrheit lose kombiniert und sie meist als eine Sache diskutiert. Ableitung und Beweis wurden als wichtiger Teil der Mathematik und Formalisierung angesehen. Das hast du noch.

Auch ist es durchaus sinnvoll, Ergebnisse abzuleiten, die sagen „ wenn diese Theorie widerspruchsfrei ist und ein Modell hat, dann …“. Die Modelltheorie tut dies seit fast einem Jahrhundert.

Was ist mit Wahrheitsprädikaten?

Aber die Leute scheinen mehr zu wollen. Sie wollen über die Wahrheit sprechen, da das eine Bedeutungsform ist, die einen besonderen Platz einnimmt. Sie unternehmen oft große Anstrengungen, um weiterhin Wahrheit und Falschheit zuzuordnen. Ein üblicher Ansatz besteht darin, Wahrheitsprädikate zu bilden – Prädikate in der Syntax, die die Eigenschaft haben, dass die Behauptung des Prädikats für eine Formel der Behauptung der Gültigkeit der Aussage entspricht (dass sie in allen Modellen wahr ist).

Beachten Sie den Schalter - ein Wahrheitsprädikat ist syntaktisch. Wir sprechen hier immer noch nicht von wahr oder falsch - der Kontext ihrer Verwendung ist immer noch, ob Aussagen mit dem Prädikat "ableitbar" oder "erhalten" sind. Theorien können mehrere Modelle haben – die meisten Theorien sind nicht nur von Dingen wie Löwenheim-Skolem kategorisch, daher können Prädikate nicht über die Wahrheit sprechen. Sie können von Validität sprechen - und darum geht es hier wirklich -, aber selbst das ist äußerst problematisch.

Unvollständige Theorien können eigentlich nichts über Gültigkeit auf die Gesamttheorie ableiten. Und tatsächlich kommt hier der Satz von Tarski über die Nichtdefinierbarkeit ins Spiel und es wird gezeigt, dass ein solches Prädikat eigentlich nicht existiert. So bleiben andere bei einer Hierarchie und Reflexionserweiterungen der Basistheorie und suchen nach einer Annäherung an einen Fixpunkt für die Gültigkeit.

Aber das hat eigentlich nichts mit der Einsicht in die Wahrheit zu tun. Es kann nicht. Es gibt nichts, was Sie tun können, um die Wahrheit zu erreichen, weil Sie nicht wissen können, ob die Theorie konsistent ist oder nicht und ob die Wahrheit existiert. Und keine Versuche, über die Ableitbarkeit hinauszugehen, geben tatsächlich ein Prädikat, das verwendet werden kann und sagen "das ist wahr". Das Prädikat ist nur nützlich, um zu sagen "das ist beweisbar".

Aber es gibt bereits Beweisbarkeitsprädikate, und diese Untersuchung ist viel profitabler. Wahrheitsprädikate sind stimmlose Orakel. Sie helfen niemandem dabei, Behauptungen über die Wahrheit aufzustellen. Sie sind einfach Neuformulierungen von „wenn wir wüssten, dass X konsistent ist, und wir einen platonischen Blick hätten, der die Wahrheitswerte in allen Modellen sehen könnte, und wir könnten die unendlichen Möglichkeiten zusammentragen und die für immer verborgenen Gültigkeiten sehen, dann würde dieses Prädikat darauf angewendet Klasse von Aussagen würde mit den gültigen Behauptungen übereinstimmen". Aber wenn wir diesen übernatürlichen Anblick hätten, könnten wir leichter einfach sagen: "Hey, das stimmt in diesem Modell - und das dort drüben ist falsch." Ohne das können wir das Prädikat verwenden, um zu sagen "Wahrheit ist in dieser Ableitung bewahrt". Was nichts hinzufügt.

Ein Wahrheitsprädikat spricht nicht über Wahrheit. Es ist für den Punkt irrelevant.

So...

Also... das Leben geht weiter. Mein einziger Punkt beim Posten dieser Antwort war, zu veranschaulichen, dass die ursprüngliche Frage eine gemeinsame offensichtliche Annahme war, die tatsächlich falsch ist. Sie sollten nicht in der allgemein verwendeten Ambient-Theorie über die Wahrheit sprechen - sprechen Sie einfach über das, was beweisbar ist, und Sie sind in Ordnung. Wenn Sie über die Wahrheit sprechen wollen, stellen Sie sicher, dass Sie die Ambient-Theorie spezifizieren, und es ist eine, in der solche Diskussionen sinnvoll sind. Oder sprechen Sie von bedingten Modellen, wie es Modelltheoretiker tun.

Es mag für manche Menschen intellektuell nicht befriedigend sein. Während ich dies schreibe, hat meine Antwort eindeutig drei negative und zwei positive Stimmen erhalten, sodass sie bei einigen anonymen Lesern einer Mathematik-Website nicht richtig ankommt. Aber an dem Punkt gibt es nichts Kontroverses. Es ist seit fast 100 Jahren bekannt und es ist immer noch ein häufiger Irrtum.

Ich denke, dass die dritte weder offensichtlich wahr noch offensichtlich falsch ist.
@AsafKaragila: Ich auch. Ich habe es dort hingestellt, weil es eine berühmte Anekdote über zwei Mathematiker gibt, die es für offensichtlich hielten, aber auf beiden Seiten. Es diente eher dazu, den Punkt zu veranschaulichen, dass viele den Kategoriefehler machen, Theoreme "wahr oder falsch" zu denken, anstatt "in Umgebungssystem S zu erhalten". Wahrheit oder Falschheit, ja sogar Gültigkeit, sind nicht wirklich anwendbar, wenn Ihr umgebendes System keine Semantik liefern kann, auf der diese Behauptungen aufgestellt werden können.
Ihr dritter Absatz ist in seiner präzisesten Form Tarskis Undefinierbarkeitssatz. Aber da ich mir bewusst bin, was es ist, bin ich immer noch nicht einverstanden mit den beiden vorangegangenen Absätzen. Relativ zu einem anderen System kann man von Wahrheit sprechen, zum Beispiel ist arithmetische Wahrheit in ZFC definierbar (aber nicht in PA). Die Wahrheit im mengentheoretischen Universum ist in ZFC nicht definierbar, aber na und?
Wenn Sie glauben, dass es ein mengentheoretisches Universum gibt und die Logik erster Ordnung seine Wahrheit erfasst (was bedeutet, dass Axiome wahr sind [was auch immer das bedeutet]), dann können wir beweisbar/widerlegbar als wahr/falsch interpretieren. Warum müssen wir in der Lage sein, (formale) Wahrheiten in dem Universum zu definieren, von dem wir annehmen, dass es existiert? Wenn es Sie stört, über Wahrheit und Falschheit zu sprechen, können Sie die Frage und alle Antworten (von denen Sie zu behaupten scheinen, dass sie fehlerhaft sind) noch einmal als "beweisbar wahr" und "beweisbar falsch" lesen.
Eine letzte kurze Bemerkung: Formale Wahrheit im mengentheoretischen Universum ist tatsächlich definierbar, aber nicht einheitlich. Dies bedeutet, dass Sie, wenn Sie die Komplexität Ihrer Formeln (z. B. in der Levy-Hierarchie) einschränken, die Wahrheit für diese Formeln definieren können (unter Verwendung einer einzigen Formel mit größerer Komplexität). Dass Sie dies nicht auf einen Schlag (mit einer einzigen Formel) tun können, sollte Sie nicht glauben lassen, dass mathematische Wahrheiten keinen Sinn machen oder wahr/falsch eigentlich bedeutungslos sind.
@Burak: Mein Kommentar gab zwei Gründe an, warum die vorhergehenden Absätze falsch waren. Ja, das erste war die Standard-„Konsistenz, die in derselben Theorie für Theorien nachgewiesen wurde, die stark genug für PA sind“, um darauf hinzuweisen, dass die einfache Annahme der Standard-Ambient-Theorie nicht ausreicht, um diese Behauptungen aufzustellen. Der zweite Grund bestand jedoch darin, darauf hinzuweisen, dass sich die Vorstellung davon, was wahr oder falsch ist, basierend auf der Ambient-Theorie dramatisch ändert, wenn Sie nicht von der Standardumgebung ausgehen. Keine dieser beiden Aussagen ist umstritten, und sie decken das ab, was ich zeigen wollte.
@ex0du5: Ich sehe, was du zeigen willst. Andererseits macht eine Änderung der Hintergrundtheorie, die sich auf die formale Wahrheit auswirkt, weder die Frage bedeutungslos noch eine der Antworten. Sie können einfach davon ausgehen, dass die Frage des OP in ZFC als Hintergrundtheorie stattfindet (oder eine Theorie, in der er Objekte seiner Theoreme formalisieren kann). Es könnte mein Missverständnis Ihrer Antwort sein, aber ich habe das, was Sie geschrieben haben, als äußerst skeptischen Ansatz interpretiert, bei dem Unvollständigkeits- / Undefinierbarkeitsphänomene verwendet werden, um jede Art von semantischem Begriff abzulehnen.
@Burak: Es gibt viele Möglichkeiten, ein Programm zu starten, um "dies zu reparieren". Aber es ist wichtig, sich darüber im Klaren zu sein, was diese Programme versuchen. Beweisbarkeit und Wahrheit sind getrennt und wurden in metamathematischen Untersuchungen schon vor Gödel klar abgegrenzt. Wir können der Beweisbarkeit eine Modalität geben und von zugänglichen Beziehungen sprechen, aber die hier zu verwendenden Begriffe sind „ableitbar“ oder „erreichbar“. Gehen Sie Ihren anderen Weg - Wahrheitsdefinitionen sind nicht Wahrheit. In beiden Diskussionen gibt es einen Kategoriefehler, der Syntax mit Semantik verwechselt. (Fortsetzung)...
@Burak: Eine Möglichkeit, meine Antwort zu betrachten, besteht darin, dass ich darauf hinweise, dass "es eine allgemeine Annahme ist, dass Sie in ZFC über Semantik sprechen können, wie sich Mathematiker im letzten Jahrhundert entwickelt haben", da Sie dies eindeutig nicht können. Sie können versuchen, von der Wichtigkeit der Semantik wegzusehen, Sie können die Existenz einer Semantik als Überzeugung annehmen, Sie können syntaktische Definitionen ersetzen, aber nichts gibt diese Semantik so wieder, wie sich die Mathematik entwickelt hat. Tatsächlich waren genau diese Punkte einige der Gründe, die dazu führten, dass Wahrheit und Semantik genau sein mussten.
@ex0du5: Ich verstehe. Sie wollen also auf den Unterschied zwischen der Formalisierung der Wahrheit und der „tatsächlichen Wahrheit“ (was auch immer das bedeutet) hinweisen? Habe ich recht? Wenn ja, stimme ich zu, dass es wichtig ist, die Unterscheidung im Auge zu behalten, aber da es keine Möglichkeit gibt, Metamathematik zu betreiben, ohne diese Konzepte zu formalisieren, scheint die Diskussion des Unterschieds eine philosophische Frage zu sein. Das gleiche Problem gilt auch für natürliche Zahlen. Ist es so, dass unsere Axiomensysteme arithmetisch einwandfrei sind, was bedeutet, dass das, was sie beweisen, in „den“ natürlichen Zahlen „eigentlich“ wahr ist? Gegen diese Frage ist mathematisch nichts einzuwenden.
@Burak: Ich sage nichts gegen die Formalisierung. Ganz im Gegenteil, ich sage, dass es eine klare formale Beschreibung der Wahrheit gibt, die verfeinert und zum großen Nutzen der Mathematik angewendet wurde. Meine Antwort enthält keine Philosophie. Wahrheit ist "Wahrheit in einem gegebenen Modell einer gegebenen Theorie". Der Wahrheit eine andere Bedeutung zu geben, war ein Jahrhundert lang ein Tabu, und es gibt wohlbekannte Gründe, warum dieser Fehler die Mathematiker in die Irre geführt hat. Ich beantworte wirklich die ursprüngliche Frage, indem ich einen tatsächlichen "offensichtlichen" Glauben gebe, der tatsächlich falsch ist. Ich versuche nicht, subtil zu sein.
@ex0du5: Wenn wir zustimmen, den Tarskischen Begriff der Wahrheit zu verwenden, muss ich meine ersten drei Kommentare wiederholen. Sie sagen in Ihrem Beitrag, dass es keinen mathematischen Begriff gibt, bei dem wir sagen können, "eigentlich wahr". Das ist Tarskis Undefinierbarkeit, mehr nicht. Was ich sagen will, ist, dass es, wenn Sie bereit sind, Einheitlichkeit zu opfern, einen definierbaren Begriff von "tatsächlich wahr" gibt. Ich kann Ihnen eine induktive Beschreibung wahrer Sätze geben, sagen wir in der Mengenlehre, die höchstens n unbeschränkte Quantoren haben, vorausgesetzt, Sie legen n fest. (Fortsetzung)
@ex0du5: Um unsere Missverständnisse auszuräumen, was genau ist die Aussage (in mathematischer Form), von der Sie behaupten, dass sie offensichtlich wahr ist, aber in vielerlei Hinsicht falsch ist?
Gödels Unvollständigkeitstheorem treibt mich nach ein oder zwei Bier wirklich in seltsame philosophische Diskussionen, da er besagt, dass die Regeln der Mathematik nicht auf Naturgesetzen beruhen und nur von uns 'erfundene' Werkzeuge sind: "Jede effektiv generierte Theorie, die in der Lage ist auszudrücken elementare Arithmetik kann nicht gleichzeitig konsistent und vollständig sein."
@Burak: Ich habe meine Antwort erweitert. Ich hoffe, es klärt auf, warum Teilprädikate niemals die Ontologie der Wahrheit sprechen können und wie syntaktische Konstrukte nichts dazu beitragen, den Punkt der unzugänglichen Semantik von Theorien anzusprechen, die stark genug für die Gödel-Konstruktion sind. Wenn nicht, weiß ich nicht, was ich sagen soll, da dies sehr bekannt und kaum kontrovers ist, obwohl ich verstehe, dass es bei einigen Menschen existenzielle Angst zurücklässt.
@aRestless: Ich denke, Ihr Kommentar zu den Naturgesetzen drückt einen wichtigen Teil dessen aus, warum dies bei so vielen einen so starken Eindruck hinterlässt. In der Natur können wir, wenn wir vollständig operativ sprechen, die Antwort auf eine semantische Frage einfach durch „Schauen“ (Durchführen der Operation oder des Experiments) erfahren. Alle Wahrheit ist erreichbar. Warum also nicht Mathe? Vor allem etwas so Einfaches wie Rechnen?
@ex0du5: Ihre Erweiterung des Beitrags hat zumindest klargestellt, wo meine Sichtweise von Ihrer abweicht. Wir können ein (formalisiertes) Wahrheitsprädikat erfinden, das uns sagen würde, ob ein (formalisierter) Satz in einem Modell wahr ist oder nicht. Sie sagen, dass diese Prädikate nicht von "Wahrheit" sprechen. Ich neige dazu zu glauben, dass sie das tun, da diese Art der Formalisierung (Einbettung semantischer Konzepte in die Syntax) von Wahrheit die einzige Möglichkeit zu sein scheint, mathematisch über Wahrheit zu sprechen (denn so wurde Mathematik lange Zeit syntaktisch in a formelles System).
@ex0du5: Ein Nebenkommentar zu Ihrem Absatz "das Problem". Selbst für den Fall, dass unsere Theorie ihre eigene Konsistenz beweist (oder äquivalent, dass sie ein Mengenmodell hat), könnten Sie nicht über die Wahrheit eines Satzes sprechen, da Sie leugnen, dass Wahrheitsprädikate über Wahrheit sprechen. Andernfalls könnten alle Ihre Probleme über ein Axiom gelöst werden, das die Existenz eines Modells Ihrer Theorie behauptet. Abgesehen davon bin ich nicht sicher, ob es "bekannt" ist, dass Wahrheitsprädikate (oder jede Formalisierung semantischer Konzepte in der Modelltheorie) tatsächlich bedeutungslos sind.
@ex0du5 Sie haben geschrieben: "Eine Theorie, die stark genug ist, um die Gödel-Diagonalisierung auszudrücken, kann niemals ihre eigene Konsistenz beweisen". Ist diese Aussage „wahr“?
@VladimirReshetnikov Ich weiß es nicht. Aber ich weiß, dass es in diesen Theorien beweisbar ist.
@ex0du5 Du hast geschrieben: "es ist in diesen Theorien beweisbar". Anscheinend ist dies eine Aussage, die allgemein über bestimmten Theorien quantifiziert wird. Stimmt diese Aussage wirklich?
@ex0du5 Ich nehme an, Sie haben die Beweisbarkeit in all der unendlichen Vielzahl von Theorien, über die Sie quantifizieren, nicht manuell überprüft. Wie können Sie sagen: „Ich weiß, dass es in ihnen beweisbar ist“, ohne der Aussage, die die Beweisbarkeit in einer Theorie ausdrückt, irgendeinen Wahrheitswert zuzuschreiben? Etwas zu wissen bedeutet zumindest, dass man glaubt, dass es wahr ist, nicht wahr?
@VladimirReshetnikov Es tut mir leid, dass meine Antwort unbeschwert war. Deine Ausgangsfrage habe ich genauso übernommen. Aber ehrlich gesagt denke ich, dass die einzigen Grundlagen, die sich dieser Frage ernsthaft nähern, radikale Konstruktivisten irgendwo zwischen Prädikativismus und Ultrafinitismus sind, wo eine solche Quantifizierung ungültig ist. Ich persönlich habe eine große Affinität zu den Computational Foundations in diesen Schulen und ihrem starken Fokus auf Semantik, da ich denke, dass dies der einzige Ansatz für Sinnhaftigkeit ist, der funktionieren kann. Ich habe versucht, das oben zu vermeiden und mich an Dinge zu halten, die allgemein akzeptiert werden, aber vielleicht lässt es sich nicht vermeiden.

Folgende Aussage ist falsch:

The inner angles of a triangle always sum to 180 degrees.

Während es plausibel klingt, dass die Summe der Winkel eine Konstante ist, ist sie tatsächlich eine Eigenschaft des Raums. Im euklidischen Raum ergeben die Innenwinkel eines Dreiecks immer 180 Grad.

Aber es ist ziemlich offensichtlich und es ist wahr.
@ bjb568 Ich denke, Robert denkt vielleicht an nicht-planare Dreiecke.
@MJD Nun, das macht mehr Sinn, aber dann ist es nicht offensichtlich ...
Ja, es scheint offensichtlich, dass die Summe der Winkel eine Konstante ist. Aber eigentlich gilt das immer nur für ebene Dreiecke.
Es scheint mir nicht offensichtlich, dass die Winkel eines ebenen Dreiecks immer die gleiche Summe haben. Tatsächlich scheint es erstaunlich, und ich bin sicher, Sie könnten andere Stellen auf dieser Website finden, wo andere Leute sagten, es schien erstaunlich zu sein.
Diese vielbeachtete Antwort beginnt beispielsweise mit „Ich fand es total erstaunlich, dass sich die Winkel in einem Dreieck immer zu 180 Grad addieren“.
Ich sehe, dass die Richtigkeit der Behauptung nicht sofort offensichtlich ist. Es ist auch ziemlich pingelig darauf hinzuweisen, dass seine Korrektheit nur in der euklidischen Geometrie bewiesen wird. Aber ich dachte immer, es sei zumindest leicht zu sehen, da offensichtlich die Außenwinkel eines konvexen n-Ecks 360 + 180 n Grad betragen. Daher war ich verblüfft, als ich von nichteuklidischen Geometrien erfuhr. Jetzt bin ich mir nicht sicher, was ich mit dieser Antwort anfangen soll. Soll ich es bearbeiten/löschen oder einfach so lassen wie es ist?
Nun, wenn wir das Wort Dreieck ein 2d, 3-seitiges Polygon lassen. Dann ja, denn wenn wir die Längen eines Liniensegments erhöhen, muss man auch vergrößern. Denken Sie darüber nach, wenn dies nicht wahr wäre, würde es bedeuten, dass ein Dreieck zwei stumpfe Winkel haben könnte, da sich die Eigenschaft aller inneren Winkel auf 180 Grad summiert würde erfordern, dass bei einem Winkel von 91 Grad die anderen 89 Grad ergeben, was nicht stumpf ist. Ich sehe, Sie zeigen auf eine besondere Art von Raum, aber weil der Raum, in dem wir leben, so funktioniert, wie er funktioniert, summieren sich die Innenwinkel eines Dreiecks auf 180 Grad ... (ich werde weiter unten fortfahren.)
Fortsetzung ... Dies auch, weil ein Dreieck Eigentum unseres Raums ist, nach Ihrer Definition, weil die Dreieckslängen erweitert und verkürzt werden müssen, um die Dreieckswinkel zu ändern, was es zu keinem Polygon machen würde, da es eine offene Form sein müsste.

Wenn ein Aussagenkalkül A alle Sätze des Aussagenkalküls B unter Ablösung und gleichförmiger Substitution für Aussagenvariablen enthält, B aber nicht alle Sätze von A enthält, dann ist eines der kürzesten einzelnen Axiome von A länger als eines der kürzesten einzelnen Axiome von B. Oder man könnte willkürlicher sagen: "Wenn der Aussagenkalkül A größer ist als der Aussagenkalkül B, dann ist eines der kürzesten einzelnen Axiome von A länger als eines der kürzesten einzelnen Axiome von B."

Haben Sie das falsche "Theorem" angegeben, oder ist dies eine wahre kontraintuitive Aussage? (Offensichtlich bin ich nicht wirklich intuitiv irgendetwas :-) )
@MarkHurd Es ist ein "falsches Theorem". Der reine Implikationskalkül hat ein einzelnes Axiom mit 13 Buchstaben, aber die einzelnen Axiome für beispielsweise BCI sind länger.
Viel Glück bei der Suche nach einem Laien, dessen Augen beim Begriff "Aussagenkalkül" nicht glasig werden, geschweige denn einem, der hier etwas Offensichtliches findet.
@rumtscho Ich arbeite nicht in der Wissenschaft. Ich habe auch nur einen Logikkurs am College und 2 Analysiskurse belegt.

Vielleicht ist das nicht ganz das, wonach Sie gesucht haben, aber es macht trotzdem Spaß! Wie wäre es mit einem Beweis, der offensichtlich falsch ist, aber (für Neulinge) ist es schwierig herauszufinden, was falsch ist.

Lassen X = j . Dann

X 2 = X j
X 2 j 2 = X j j 2
( X + j ) ( X j ) = j ( X j )
X + j = j
2 j = j
2 = 1

ja, man kann alles Mögliche "beweisen", wenn man durch 0 dividiert ... ;)

Ein "offensichtlich wahrer" Satz:

Wenn Sie ein 3D-Objekt nehmen U und zerhacke es in endlich viele Stücke, irgendein Objekt v Ich ordne diese Teile so an, dass sie das gleiche Volumen wie das Objekt haben U Ich begann mit.

Aber tatsächlich sagt uns das Banach-Tarski-Paradoxon, dass dies nicht wahr ist – wenn wir unsere endlich vielen Teilmengen von konstruieren U „komisch“ genug, wir können tatsächlich einen bauen v mit jedem Volumen, das wir möchten.

Jede stetige Funktion ist zumindest irgendwo differenzierbar, oder?

Falsch, die Wierstraß-Funktion ist ein berühmtes Gegenbeispiel

https://en.wikipedia.org/wiki/Weierstraß_Funktion

Sie ist überall stetig, aber nirgends differenzierbar.

Eine analytische Funktion mit kompakter Unterstützung verschwindet identisch.

Das stimmt tatsächlich, solange wir drinnen bleiben C N bzw. R N . Es wird falsch, wenn man eine kompakte analytische Mannigfaltigkeit nimmt, aber dann ist es offensichtlich, dass es falsch ist.

Lebesgue sagte einmal, dass die Projektionen von Borel einsetzten R 2 Auf einer seiner Achsen befinden sich auch Borel-Mengen. Diese Tatsache ist eigentlich falsch, deren Erkenntnis dem kurzlebigen Mathematiker Michail Jakowlewitsch Suslin zugeschrieben wird.

Leider ist es sehr schwierig, ein Gegenbeispiel zu finden. Die einzige, die ich je gesehen habe, bringt ein Ergebnis in der deskriptiven Mengenlehre zustande N N und nutzt die Tatsache, dass es homöomorph zu ist R .

Ich hoffe, Sie erinnern sich falsch oder missverstehen die Argumente, die Sie gesehen haben, weil N N ist nicht homöomorph zu R . Zum Beispiel ist einer verbunden und der andere nicht. Einer ist σ -kompakt und der andere nicht.
Das hängt von der Topologie ab, aber es könnte sein, dass das Argument, an das ich denke, den Raum aller Folgen (sowohl endlicher als auch unendlicher) natürlicher Zahlen verwendet oder den Homöomorphismus unter Ausschluss von entwickelt hat Q .
"Das hängt von der Topologie ab". Offensichtlich. "Der Raum aller Folgen (sowohl endlicher als auch unendlicher) natürlicher Zahlen." Vielleicht, aber das ist es nicht N N . „Mit Ausnahme von Q ." Aber das ist es dann doch nicht R . Ein oder zwei Worte zur Klärung der Ungenauigkeit wären jedenfalls nicht überflüssig.
@jjfunk: Ist es möglich, dass die Karte, von der Sie sprechen, der Homöomorphismus zwischen dem Baire-Raum und den Irrationalen ist, die durch unendliche fortgesetzte Brüche gegeben sind?

Der Traum des Neulings:

( X + j ) P = X P + j P
.

Offensichtlich falsch. Aber wahr in der Charakteristik P .